1st series [1] [2] [3] [4] [5] [6] [7] [8] [9] [10] [11] [12] [13] [14] [15] [16] [17] [18] [19] [20] [21] [22] [23] [24]  2nd series [1] [2] [3] [4] [5] [6] [7] [8] [9] [10] [11] [12] [13] [14] [15] [16] [17] [18] [19] [20] [21] [22] [23] [24] [25] [26] [27] [28] [29] [30] [31] [32] [33] [34] [35] [36] [37] [38] [39] [40] [41] [42] [43] [44] [45] [46] [47] [48] [49]

  View the latest questions and answers at askaphilosopher.org

Ask a Philosopher: Questions and Answers 45 (2nd series)

When referring to an answer on this page, please quote the page number followed by the answer number. The first answer on this page is 45/1.

(1) Demetreus asked:

Descartes and Plato equate knowledge with complete certainty.

Do you agree that knowledge requires this very high standard?

If knowledge means being certain, is there anything we can
truly know?

---

Methinks Demetreus would not have posted this question, if it hadn't been on his philosophy class assignment sheet. Quite a lot of these questions find their way to Ask a Philosopher. I'm not averse to answering them because they show something revealing about the way philosophy is taught in many colleges and universities.

The topic is Epistemology or 'Theory of Knowledge'. The term comes from the Greek word episteme which Plato in his Republic contrasts with doxa or mere belief. According to Plato, you can't have episteme of things in the empirical world — the world in space and time — because empirical objects are shifting and uncertain. You can only have doxa. Whereas episteme is reserved for eternal things: the objects of mathematics, and, ultimately, the Forms.

Descartes argued in his Meditations that empirical knowledge is possible, but only on the basis of a proof of the existence of a benevolent God, who has so arranged things in this world that provided that we use our capacity for judgement responsibly, we will not be led astray. Even so, Descartes knew full well — as he explains in Meditation 6 — that even when you exercise exquisite care in making judgements, a judgement can still turn out to be false. The acquisition of empirical knowledge relies on mechanisms like perception, which for natural reasons — the laws of nature which God Himself decreed — can sometimes fail to deliver the goods.

Why can we only know the Forms? Why is God needed to make knowledge possible? These are deep and fascinating questions for students of Plato and Descartes. Unfortunately, a style of lazy thinking seems to have crept into Epistemology, which lays the blame on the requirement of certainty. According to the lazy view, Plato and Descartes were wrong, because they didn't realize that certainty isn't required for knowledge!

This is such a preposterous idea, no wonder generations of tyro philosophy students are baffled by it.

The anonymous author of the question, an instructor at some college somewhere, has obviously realized that there is a lacuna here so he/ she has inserted the adjective 'complete'. This is like a flashing red light. Why the need for a qualification? Certainty is certainty. Does knowledge entail that you are certain, or does it not? If I tell you I'm certain, and then you go on to ask me if I'm completely certain, you deserve a smack in the jaw.

Most persons who have not been subjected to first-year Epistemology classes would say knowledge does imply certainty. Suppose I remark, 'Sue knows that Bob is cheating on her.' I have made two claims. First, Bob is cheating on Sue. That's bad. It's not the sort of thing you'd want to make a mistake about. (I haven't said I know that Sue knows that Bob is cheating on her because that's already implied by what I said.) The second claim is that Sue knows this. By implying that I know, I have also implied that I am sure of my ground. I am certain. By stating that Sue knows, I imply not only that Sue is sure of her ground, but also that she in a position to be sure. If Sue told someone, 'Bob is cheating on me', the hearer could take this information as authoritative, not open to doubt.

That's how he concept of knowledge works. If you are not sure, if there is any element of uncertainty, then you should say so otherwise you are behaving irresponsibly. You are giving your audience grounds for thinking that you are an authoritative source of the information in question, when you are not. You are only guessing.

Do we really go through all this palaver in daily life? Yes, we do. We just don't think about it in such explicit terms.

However, these are just the kinds of facts that the sceptic exploits. Arguments for scepticism typically take the form of pointing out the many ways in which it is conceivable that you could be wrong. Descartes in Meditation 1 comes up with a real show stopper: all my assumed 'knowledge' of the external world could just be a dream fed to me by an evil demon. You might think the idea is spectacularly improbable, but it is logically possible. In that case, you can't rule it out completely. You don't have the right to be certain after all.

However, it is not necessary to go to such extremes in order to raise a question mark about ordinary claims to knowledge. 'Is Bob cheating on Sue?' 'Yes, I saw him together with Mary.' 'Does Bob have a twin brother in Australia?' 'Search me if I know.' 'If Bob had a twin brother, wouldn't it be possible that it was his twin brother on a visit from Australia you saw with Mary?' 'Yes, I suppose so.' 'In that case, would you like to revise your statement?'

It doesn't require too much ingenuity to come up with suitable defeating questions for just about any knowledge claim.

Philosophers have offered various solutions. According to David Lewis, knowledge is a contextual notion. Assume that Bob is cheating on Sue. The question is whether I know this. Based on what I saw them do in the park, there's no question in my mind. I know. I'm certain that he is. Then you hit me with the question about the possible twin brother. Now I don't know. All you did was ask me a question! (As it happens, Bob doesn't have a twin brother. But of course that's irrelevant because I've never thought to investigate.)

Lewises solution grabs the horns of the dilemma in both hands, but I don't like it because it leaves the whole notion of 'knowledge' seeming too damn paradoxical. I don't have a better solution. All I know is saying 'knowledge doesn't entail certainty' (or 'complete certainty' if you will) is a complete cop-out.

Geoffrey Klempner

back

(2) Dave asked:

I have a very naive question. What do people mean when they talk about absolute vs. relative truth? I study math, and in math a statement is either true or not true what would it mean for a statement to be relatively true or absolutely true?

What is an example of a statement that is relatively true but not absolutely true? In common usage, it seems that people use 'relative truth' to mean a statement P that is true only some of the time. But then wouldn't the statement 'P is true some of the time' be an 'absolute truth?'

---

Yes, you are correct. 'Relative truth' usually means that something is true for you if you believe it, and false for you if you do not believe it. You can then ask whether all truth is relative, or not. If it is, then the statement 'All truth is relative' is absolutely true, in which case not all truth is relative; and if it is not absolutely true than that means that some truth is absolute, in which case not all truth is relative.

Helier Robinson


back

(3) David asked:

I was wondering if you could tell me if I have a valid criticism of a version of the moral argument for the existence of God.

The argument was formulated as:

1. If God doesn't exist, then objective moral standards don't exist

2. Objective moral standards exist

Therefore, God exists.

This is a valid syllogism, so if one could prove the premises one would have a proof. But suppose you try to prove premise 2. You can't invoke God to do it, as you're trying to prove God exists. So suppose you could prove 2 without invoking God. Then you've shown that objective moral standards exist regardless of God's existence (which undercuts premise 1).
So, it seems to me, there's no hope of proving premise 2 without sinking the argument. Is this criticism valid?

---

Yes, it is. An equivalent form of 1 is 'If moral standards exist then God exists' and then the validity of the argument is clear. But neither 1 nor 2 is obviously true, so, although valid, the argument is not sound.

Helier Robinson


back

(4) Gordon asked:

hi what do philosophers think of psychoanalysis especially Carl Jung's method. My reasons for asking this question are, will therapy with a jungian analyst help me with my existential angst? Or is their any other alternatives you can suggest?

---

It's no use asking what philosophers think about anything, because they always disagree if you ask enough of them. I myself admire Freud for opening a new way of understanding the mind, even if he was largely wrong in his details; and I admire Adler and Jung as well. Who can tell whether any form of analysis would help your angst? Try, and see if it works. It does not necessarily have to be Jungian.

Helier Robinson


back

(5) Dave asked:

I have a very naive question. What do people mean when they talk about absolute vs. relative truth? I study math, and in math a statement is either true or not true what would it mean for a statement to be relatively true or absolutely true? What is an example of a statement that is relatively true but not absolutely true? In common usage, it seems that people use 'relative truth' to mean a statement P that is true only some of the time. But then wouldn't the statement 'P is true some of the time' be an 'absolute truth?'

---

The problem is that relative truth seems to mean different things to different people. It seems to be mostly applied to moral systems. However in general I think that the belief that moral truths are relative is as absurd as the idea that moral truths are absolute.

In general we can distinguish between contingent truths e.g.'It is raining' which can be true or false. Then there are necessary truths such as the truths of mathematics, 'Two plus two equals four' is true in all possible worlds.

However we can also distinguish certain sorts of relative truths i.e things that are only true within a presumed framework. 'James stole John's money' can only be true in a society that recognises personal possessions and has a monetary system.

Shaun Williamson

back

(6) Daniel asked:

At which point does any process become a valuable one? So far I have tried breaking down the process into parts, but can find no inherent value in any one. Conception of idea is not something we can control ideas seem to merely arrive in our heads so the idea alone is not something that has any real value. Carrying out the process doesn't give it value otherwise every idea carried out would be equal. The length of time given to the process doesn't control value as hammering a brick with a hammer for a hundred years has no more value than doing it for 30 seconds. And the culmination or product of the process... well. This is just the product of a process. It is nothing alone. if the process was worthwhile the result will be useful but only as part of a new process? I'm not sure. I know this is a rather muddled question, but im hoping for some philosophical clarity. Even just thoughts on redefining my methods if investigation.

---

Daniel you are asking the wrong questions so its not surprising that you are not getting any answers. The word 'process' is so abstract and vague and it simply doesn't have properties.

Consider these two phrases 'Doing something badly' and 'Doing something well'. Both of these describe processes but its fairly obvious which one is valuable and which one isn't. Consider the process of snapping your fingers exactly in time to the music and the process of building a good nuclear reactor. What do these things have in common beyond the fact that we can describe them both as processes.

There are no properties that all valuable processes must have in common and assuming that there are, is what has got you into this dead end. Valuable processes are valuable in their own individual ways. You might like to read about Wittgenstein's discussion of the nature of a game and what all games have in common (nothing according to Wittgenstein).

Shaun Williamson

back

(7) Gordon asked:

hi what do philosophers think of psychoanalysis especially Carl Jung's method. My reasons for asking this question are, will therapy with a jungian analyst help me with my existential angst. Or is their any other alternatives you can suggest

---

I think psychoanalysis is a pseudo science especially Jungian analysis which can only be compared to fortune telling and other superstitious practises.

However if you feel that you have problems then talking to a qualified psychotherapist can be valuable. Try to avoid those therapists who have weird beliefs and before you start therapy try to decide if you want to talk to a man or a woman. Approach the process with an open mind (don't expect that you will necessarily like your therapist). It is often easier for people to discuss their problems with a total stranger than with a friend and that is why therapy can work. Also remember that the therapist is there, not provide you with answers or to tell you what to do, they are there to help you to arrive at your own answers.

Shaun Williamson

back

(8) Bryan asked:

Where would be good school to study mereology at the graduate level? I'm not looking for any school with specifics in mind, given that I already understand the options available by wanting to find a good program in just general mereology. Thank you for your time.

---

I don't know of any schools who teach mereology (the theory of parts and wholes) at the graduate level.

Mereology is a specialist subject and although some major philosophers such as W. O. Quine have been interested in developing the subject, it has not become a part of mainstream graduate philosophy or logic courses. In fact you are the first person I have known to express any interest in the subject. It was sometimes seen as an alternative of set theory and some thinkers who rejected set theory adopted an interest in mereology instead. However axiomatic set theory has become an established part of modern mathematics, mereology hasn't.

Shaun Williamson

back

(9) Bryan asked:

Hello. I noticed that sometimes invalid argument forms can support valid arguments. For example, the syllogism:

1. No A are B.

2. All B are C.

.: 3. No A are C.

Is an invalid argument form, but can support a valid argument if A is 'cats', B is 'collies', and C is 'dogs'. Yet, I think there is something more intuitive that I am missing. My question is that does this invalid argument form warrant the valid argument above or is there a warrant that makes the 'cats collies dogs' argument still invalid? I would have thought that it would still be invalid because a cat could be a dog, but I thought that to be a poor reason since by definition a cat is not a dog.

---

It is easy to become confused about things. The definition of a valid argument is that it is one where it is IMPOSSIBLE for the premises to be true and the conclusion false.

Now the argument in your example is clearly invalid because we can think of examples where the premises are true and the conclusion is false. The fact that you have given an example where the premises are true and the conclusion is true does not matter. It is possible to find examples like that for all invalid arguments but this can never make them into valid arguments. A valid argument has to be valid no matter what the values of A,B and C are. The fact that it is valid for some special cases is not important. These special cases all depend on extra hidden premises e.g that by definition cats can never be dogs.

Shaun Williamson

back

(10) Amery asked:

I want to know if dreams can change the sort of person we are through a sort of 'life-experience'-esque way.

Is it possible?

---

I get the point of your question, Amery. Our life experience can change the sort of person we are. If our dreams are part of our life experience, then dreams can change us too. Why not?

I agree. Dreams give you ideas. They are like thought experiments, things you could never try out in the real world. Or they can shock you with the consequences of your beliefs or actions by painting the resulting scenario in lurid colours. Above all, dreams are creative. They come from us, and yet they are at the same time totally unexpected events that come out of the blue. We meet things in our dreams just as we meet things in the real world.

Our life experience includes contingencies or things that happen to us over which we have only limited control. You apply for a job. You give it your best shot at the interview. But, ultimately, it is out of your hands whether your get the job or not. And getting that job could conceivably change your life, and ultimately change you. Or you could meet that special person, fall in love, and end up emigrating to South America.

Are dreams then 'part of life experience'? in this sense?

There is a powerful argument for saying that they are not, which derives from Sigmund Freud's famous book The Interpretation of Dreams. To understand Freud's position, it is necessary to appreciate his guiding methodological assumption, which we may call psychic determinism. This is a stronger principle than determinism understood merely as the rule that 'every event has a cause'. For Freud, events in dreams have a particular kind of cause, which confers essential meaning on that event.

According to psychic determinism, every detail in your dream (that is to say, every detail that you write in your 'dream book') has a meaning and an explanation. If I dreamed that I walked into a room which had seven chairs, that is different from dreaming that I walked into a room which had several chairs. The number seven must be significant, it cannot be merely accidental. If the chairs were green, rather than just 'some colour' then that colour is significant too.

Freud held that all the details in a dream (apart from the factual content taken from real life) express wishes deriving from the subconscious, in a disguised form which has been allowed past the 'dream censor'.

In other words, your dreams are statement about you, written in code. There is nothing accidental. Everything has significance, a large part of which is sexual in nature. Of course you can be changed by your dreams in this sense, if you are able to decode that meaning with the help of a psychoanalyst. You are changed because you discover something about yourself, something that was hidden in your subconscious.

Notwithstanding the importance of dream interpretation in analytic psychotherapy, I think Freud was wrong. Many details in our dreams undoubtedly have significance, but the assumption of psychic determinism is unwarranted, even as a merely methodological principle. It is simply too strong.

My case is not merely that Freud fails to justify this principle. What Freud fails to take account of is something which is fundamental to human nature, our capacity for genuine creativity, not in the Freudian sense of 'making the subconscious conscious' but rather in the sense of producing novelty through a process which has an irreducibly random element.

There's nothing necessarily mysterious about creativity in this sense. Human behaviour would be very rigid and hidebound if we couldn't come up with novel solutions to problems. Nor is it necessary to posit some unique, indescribable 'creative faculty'. Daniel Dennett in his book Brainstorms MIT 1981, pp.296-8 describes a simple process which he terms 'generate and test', whereby candidates are randomly generated and then tested for relevance or suitability. A scientific researcher looking for a solution to a problem does this. So does a novelist. What does Katy do next? Let's try some random possibilities and see where they lead.

In dreams we do this too. One situation leads to another through a peculiar kind of 'dream logic' where the criterion of 'relevance' is applied in a very loose way. Almost anything can happen.

I talked earlier about 'contingencies or things that happen to us over which we have only limited control'. We do not only meet up with these in real life. We also meet up with them in our dreams. We create our dreams, and yet, in an important sense our dreams are also something that happens to us, which might as well have been made by the world as by our own selves.

Geoffrey Klempner

back

(11) Mel asked:

Does scientific method lead to truth.

Science laws and theories are based on evidence, and so are supported by tangible proof, but do the models correspond to what is in nature?

I am focusing particularly on gravitational scientific theories.

---

Scientific method is to look at the facts and to try and find a useful theory to explain them. Can you think of any other method that is more likely to lead to the truth? Should we invent theories that are not based on the evidence?

If you want a better theory of gravity then you must find one that fits the facts better than the existing theories do. That is how science advances.

Shaun Williamson

back

(12) Eric asked:

Does a perceived threat to ones existence give right to violation of law?

---

Your question does not make sense. Do you mean 'Does a perceived threat to ones existence justify breaking the law?'. The answer to this is no. However if someone tries to kill you then you have the right to defend yourself. Do not confuse this with 'thinking that someone MAY try to kill you'. This could just be the result of paranoia. You can defend yourself against real attacks but not against attacks that you think might happen in the future.

Shaun Williamson

back

(13) Rebecca asked:

Could you please explain what an Inverted Spectrum means?

---

A spectrum is a range, for example, of colours. Think of the range of colours in a rainbow or in a prism. Painters use a colour wheel, a type of spectrum, which shows all the colours in their relationship with one another. And in this range each colour has a complementary colour: for example, orange is the complementary of blue.

Now imagine that each person has a private view of the world in the mind that only that person can be aware of. One way of making sense of this idea is to think of pain. If I have a pain in my knee, I can go to my doctor about it. He can examine my knee for any physical problem; I can try to describe my pain as throbbing or an ache. But what cannot happen is that the doctor can actually experience my pain. The pain is private and exclusive to me. It is in my private world and no one apart from me can be in that world and have the experiences that I have.

Now think about colours. We all (apart from those who are colour-blind) use the same names for colours. If we were walking along a street and asked to point out something that was red, we would all identify a post box as an example. So there is no problem most of the time with the general agreement about what is red or green or purple.

But next consider that colour experience might be like experience of pain. Is it possible that, even though we use the same words for colours, what we actually see 'inside' as it were, is different? The idea is this. Suppose somehow I were to be able for a moment to enter into your private world and see the colours as you see them. Now we both call a satsuma orange coloured. But it is possible that if I were seeing the satsuma from inside your mind, the colour I would see I would call blue. Indeed, there might be many such differences between not just you and me but among people in general. It is conceivable that we all might be seeing our own versions of colours even though there is no disagreement among us about the names of coloured objects. Since the range of colours one person experiences might be the reverse (or an inversion) of the range of colours someone else experiences, this possibility is called the inverted spectrum.

One of the implications of the inverted spectrum possibility is in the philosophy of mind. A very widely held view in this area is functionalism. According to functionalism the mind is what exists in between input and output. Someone kicks me on the shin (input) and I cry out, 'Ouch!' (output). The pain is what is caused by the blow to my shin and the pain is the cause of my crying out. Now if we allow that there might be an inverted spectrum as described above, this functionalist explanation of what is going on is inadequate. For if you and I are asked to point out a red object (input) and we both indicate a post box (output) in functional terms there is no difference between us. But now think what the inverted spectrum implies. What is actually going on in my mind (the colour I am seeing on my internal screen, as it were) is quite different from what is going on in your mind (the colour you are actually seeing on the internal screen in your mind). So there would be a difference between us that functionalism cannot deal with.

Needless to say there is much dispute about whether or not there could be an inverted spectrum with some philosophers arguing that colours cannot be swapped around in the way suggested above. (There is some discussion of this objection in the Chapter 2, Mind in Think by Simon Blackburn published by OUP.)

Oliver Leech

back

(14) Bryan asked:

Hello. I noticed that sometimes invalid argument forms can support valid arguments. For example, the syllogism:

1. No A are B.

2. All B are C.

.: 3. No A are C.

Is an invalid argument form, but can support a valid argument if A is 'cats', B is 'collies', and C is 'dogs'. Yet, I think there is something more intuitive that I am missing. My question is that does this invalid argument form warrant the valid argument above or is there a warrant that makes the 'cats collies dogs' argument still invalid? I would have thought that it would still be invalid because a cat could be a dog, but I thought that to be a poor reason since by definition a cat is not a dog.

---

I believe you're entirely right in feeling that there's something a bit off in the idea that an 'invalid argument form' might warrant a 'valid argument.' Here's what I think it is, following your example: the 'therefore,' or '.:' is, specifically, what is not warranted. That is to say, 1. is a true statement, as indeed no cats are collies; 2., likewise, is a true statement, since all collies are dogs; and 3., too, is a true statement, since as you note no cats are dogs, by definition. But the three true statements are not meaningfully or in any way necessarily linked, and hence a valid argument has not been made.

A valid argument is composed of valid premises and a valid conclusion; what you have, however, are simply three valid premises. That is, 'No A are C' is simply not a conclusion that can be drawn from the fact that 'No A are B' and 'All B are C'--even though it may also, as here, be a true statement or valid premise in its own right. 3. is coincidentally also valid, in the case that A = cats and C = dogs, but not because 1. and 2. are valid. This is why I said above that the 'therefore' was not warranted, or has not been earned.

Hence, too, the possibility of a series of variables here that would suggest one had reached a totally absurd conclusion. A = motorcars, B = Stalin, and C= dead Russian dictators, for example, would follow the exact same (il)logic of your original example, and hence might seem misleadingly to reach a valid conclusion by invalid logic, since the final statement (no motorcars are dead Russian dictators) would be true. But let us substitute 'things or people associated with iron' for C, and the fallacy becomes apparent. Both motorcars and Stalin are associated with iron, and so the idea that 'No A are C' could serve here as conclusion is immediately absurd.

The point in this is that it literally does not matter what the variables are set to--the final statement can be totally valid OR totally invalid because it is not in any way tethered to the preceding statements or premises.

To return to your original question more fully then, the 1.2.3. series you suggested is not a valid argument with an invalid form. It is, properly speaking, not an argument at all, but rather only a series of premises that do not yet reach a conclusion. It looks like an argument because it has a 'therefore' in it, but the 'therefore' is an intruding presence.

There does seem to me, though, a really interesting strain to your question, and that is its concern with what exactly makes logic logic. Why (either ontologically or rhetorically) do we experience the 'tethering' of arguments to one another in certain (and only certain!) as being so compelling? And are there not perhaps other examples where invalid premises or fallacious argument types really do produce valid conclusions? (Keeping in mind that the example we've been working with here is that of a coincidental third statement in a series, not of a conclusion produced from the first two.) As you're thinking through these questions, one book you may find of particular interest in Jeremy Bentham's Book of Fallacies. Or, from another angle, you might find psychologist P.E. Meehl's essay, 'Why I Do Not Attend Case Conferences,' useful; collected in his Psychodiagnosis: Selected Papers, it starts off with some general sniping and (from my perspective) unnecessarily snarky commentary on his profession, but from there it quickly moves through a very helpful list of common fallacies in social science arguments and explanations of same.

I hope this was helpful, and encourage you to continue in your thoughtful questioning.

Ira Allen

back

(15) Bryan asked:

Hello. I noticed that sometimes invalid argument forms can support valid arguments. For example, the syllogism:

1. No A are B.
2. All B are C.
.: 3. No A are C.

Is an invalid argument form, but can support a valid argument if A is 'cats', B is 'collies', and C is 'dogs'. Yet, I think there is something more intuitive that I am missing. My question is that does this invalid argument form warrant the valid argument above or is there a warrant that makes the 'cats collies dogs' argument still invalid? I would have thought that it would still be invalid because a cat could be a dog, but I thought that to be a poor reason since by definition a cat is not a dog.

---

You need to distinguish between a syllogism which allows a true consequent and one that requires a true consequent. If it does not require it, then it is invalid. Your example, AeB, BaC, AeC allows a true consequent but does not require it, and your example of cats-collies-dogs shows this. An example of it being clearly invalid is: No cats are bitches, all bitches are female, therefore no cats are female.

Helier Robinson


back

(16) Nicky asked:

Does it make sense to talk about something that doesn't exist, and if so, why?

---

Yes, it makes sense in a number of ways. For example, in the early days of modern science fire was explained by means of a concept called phlogiston: it was supposed that flame was escaping phlogiston. But then it was found that phlogiston, if it existed, had to have negative mass; so the theory of fire being combination of fuel and oxygen at high temperature replaced the phlogiston theory. But the non-existence of phlogiston did not make the theory senseless — just false. So also with the theory of the electromagnetic ether. Non-existent entities also exist in myth and metaphor, but do not thereby make these senseless.

Helier Robinson


back

(17) Ben asked:

If I feel jealous, my evil side tries to surface. It tries to corrupts my inner balance, happiness and switch my focus. I fight this evil through a great struggle because I do not want to cause myself or anybody any harm. So where does jealousy come from? and how does it go away?

---

You said it yourself: it comes from you evil side. Why you have an evil side is another matter. Nobody has ever properly explained where evil comes from, but evil is a fact of life , and we have to fight it. The more we succeed in that fight the more mature we become, and with maturity, jealousy diminishes and eventually goes away. By the way, many people confuse jealousy and envy, so maybe you have envy in mind. Jealousy is a special case of envy, namely envy of the love or attention that another person is receiving.

Helier Robinson


back

(18) Mel asked:

Does scientific method lead to truth.

Science laws and theories are based on evidence, and so are supported by tangible proof, but do the models correspond to what is in nature?

I am focussing particularly on gravitational scientific theories.

---

Yes, they must correspond to nature; if they didn't then it would be meaningless to speak of their truth (or, more accurately, of the probability of their truth.) Probably the most significant feature of (mathematical) theories is that they predict empirical novelties, successfully and often; if they were not true, or largely true, the success of these predictions would be pure chance and so very rare.

Helier Robinson


back

(19) Jim asked:

What is the philosophical definition of 'time'? It may seem simple but try to give an answer to a bright 10 year old boy.

He can read a clock. He wants a simple explanation of 'time' if there is one. I have come to the belief there isn't.

Geographically, us rounding the sun, is only a definition of the measure of time.

---

Jim, I will accept your challenge of explaining time to a 10 year old. So there will be no discussion of theories of time from Aristotle to the present day, no arguments for or against the reality of time, no examination of fatalism and the problem of future contingents, or temporal becoming and the myth of passage, or any of the other stock problems from the academic philosopher's toolbag.

These are all gripping problems for me, but I would be struggling to explain why they are gripping to a 10 year old.

Time is a problem which baffles and mystifies me, and probably (though I can't be sure) was one of the first philosophical questions which I ever thought about, long before I had ever heard of a subject called philosophy.

I have a memory fragment as a boy of being driven by my father to the dentist. This was before they had high speed drills and pain-killing injections. I would have been about 9 or 10. I know this because I remember consoling myself with the thought that in an hour I would be travelling home in the car and I would say out loud, 'Cool for Cats!' And sure enough, on the way home I did remember to say it.

Cool for Cats was the title of a British popular music TV programme for young people which according to Wikipedia ran from 1956 to 1961. In 1961, I was 10 years old.

That little episode illustrates as well as anything the mystery of time. I said 'Cool for Cats!' twice. The first time I said it, the pain of the dentist's drill was in the future. The second time I said it, the pain was in the past. What a difference an hour makes! Yet now, both events are just things that happened a long time ago.

A clock measures the distance between events or 'things that happen' just as a ruler measures the distance between two points. There's no mystery about that. The hands of a clock go round the dial at a standard speed which is the same for all clocks, measured in hours, minutes and seconds. A ruler is a standard distance measured in inches or centimetres.

It would be a rather strange request if you asked, 'Don't tell me about how you measure length, I just what to know what length is!' Length just is what a ruler measures, there's nothing more to it. (Well, that's not strictly true if you really wanted to delve into the nature of space, but it will do for now. I would argue that there's no mystery about length or distance, the way there is about the passage of time.)

A clock is an instrument which measures events, and your entire life up to the present moment — or indeed the history of the universe — is just a series of events. When I said 'Cool for Cats!' twice, those were just two events in my life, two insignificant events in the history of the universe. Yet at the time there was a world of difference between them.

But how would you express that 'difference' in the form of a definition? Consider: every second that the clock ticks is an example of the ever-shifting difference between past, present and future. Sometimes, an event (like a trip to the dentist) brings the difference to our attention. But it was there all the time. It is here now, as I type these letters on the screen, as I look away from the computer and look back again, as I lift my finger to scratch my nose, or take a sip of lukewarm coffee I made half an hour ago.

Maybe that's one way philosophers stand out from other people: they learn to ponder things — seemingly trivial things — you wouldn't normally think about. You could describe it as a sense of 'childlike wonder', although not all children are equally gifted with it.

I don't know if this is making any sense. Or whether it would make sense to a 10 year old. I'm not talking about fancy theories. I'm talking about an experience, something that you just have to see. And when you see it, see it for what it is, you realize that you don't understand it at all. That's why I call time a mystery.

Geoffrey Klempner

back

(20) Ken asked:

what is the nature of reality?

---

There are three common definitions of reality. One is that reality is all that exists independently of being perceived — i.e. it exists whether it is perceived or not. Another is that reality is all that we perceive around us provided that it is not illusory. And the third is that reality is what makes true statements true. Many believe that these are all equivalent, but it can be shown that they are not. There is not space enough here to do this, but if you are interested try downloading my (free) book 'Belief Shock' from www.sharebooks.ca.

Helier Robinson


back

(21) Mark John asked:

Are technological advancements, moral or immoral?

---

Neither. It is the uses to which they are put that are moral or immoral, to varying degrees.

Helier Robinson


back

(22) Sinead asked:

Can acts of terrorism be justified?..with reference to ted Honderich's concept of'terrorism for humanity'.

---

Many learned authorities defend the indefensible, and the point of referring to them sometimes escapes me.

Nevertheless, because it bears on the moral consequentialism discussed in another answer, here goes.

Ted Honderich has written a great deal in a certain contrarian vein, the common thought of which is that everything Bush knows about the evils of islamic terrorism is wrong. There is hope that this contrarian trend in thought thought may die a death now that Bush has been replaced by Obama, because it is never a good idea to assess something on the basis if whether a politician you hate is against it.

In this vein, Honderich wrote a piece called 'Terrorisms in Palestine', which as the name implies, argues that Hamas is no worse than the Israeli Government. What ought to interest us is the philosophical argument that underlies this conclusion.

Partly because I feel under-qualified in historical fact, and partly because on experience I am sceptical that arguments between variant interpretations of the facts can produce any general understanding or conciliation, I am just going to skip the history of the Middle East in what Ted Honderich has to say about terrorism — or at least, I'm not about to argue variant interpretations of the facts, and I will offer no verdict here on what ought to happen in Palestine. This might seem like cheating, except that it seems to me that there is more than history and politics to Ted Honderich's thought here. There is also, it would seem, the application to history of some rather striking moral thought, and it may be useful to examine that thought on it's merits.

TH opens 'terrorisms in palestine' with a statement of his guiding principal — a statement which is recognisably philosophical. He pursues precision and clarity, and he uses lots of clauses. He writes:

'My principle... the Principle of Humanity... is that the right or justified thing as distinct from others — the right action, practice, institution, government, society or possible world — is the one that according to the best judgement and information is the rational one in the sense of being effective and not self-defeating with respect to the end of getting and keeping people out of bad lives — which lives can be clearly defined in terms of fundamental human desires or goods. In my list there are six such desires or goods — a decent length of conscious life , a bodily quality of life including absence of pain, freedom and power in various settings, respect and self respect, and the goods of culture, including knowledge, religion and more.'

Clarity might be helped by brevity, so here's my brief precis of the above:

'an act is right in so far as the foreseeable consequences prevent bad lives'

You should compare this with the quoted paragraph. The profit in the precis is that, put like this, we can see that the two crucial ideas in TH's 'Principle of Humanity' are bad lives and foreseeable consequences . The precis isn't perfect, because it is imprecise on one rather important point, a point of imprecision also evident in TH's own expression of the 'Principle of Humanity'. It's a fair guess that TH does not have it in mind that in order to get and keep people out of bad lives it is obligatory to go around killing anyone who may now have, or be likely in the future to have, a bad life . No, we can assume that TH wants to 'get and keep' people out of bad lives in the sense in which the project in fact amounts to getting people into good lives, and keeping them there. We should therefore amend the precis so that if TH wants to prevent bad lives, as my precis has it, then this is preventing bad lives in some sense in which this is synonymous with promoting good lives. Therefore, even though TH makes no mention of good lives, it is still the case that a better precis of his 'Principle of Humanity' would read:

'an act is right in so far as the foreseeable consequences promote good lives'

A reservation is that it might be somehow important that TH fights shy of the expression 'good lives' in expressing his 'Principle of Humanity'. I suppose the implication of TH's choice here is that it may be profitably clear what a bad life is, whereas the age old pursuit of a definition of the good life has — it might be thought — merely stirred up mud. Consider the utilitarian who is asked to define happiness. And when television shows you a mother grieving by her son's coffin the idea that unhappiness is something much clearer than the slippery idea of happiness is hard to avoid. Here is rock bottom. Here is a clear vision of hell. Well, I can see the justice in that thought. At the same time I wonder whether it goes the distance. Do such clear intuitions of another's suffering carry over into clarity about exactly which lives are bad lives? Are we left with the kind of notion of a bad life which can do the work TH wants it to do, namely the work of distinguishing right actions from wrong ones?

Despite our moments of clear vision on the TV, TH feels a philosopher's need to define'bad lives'. He goes about this by listing some fundamental human goods that bad lives may be understood in terms of.

'In my list there are six such desires or goods — a decent length of conscious life , a bodily quality of life including absence of pain, freedom and power in various settings, respect and self respect, and the goods of culture, including knowledge, religion and more.'

That's quite a good list of things that matter to humans. An adequate definition would go beyond any such list, however. As things stand I do not yet know whether a bad life , such as a right action is concerned to prevent, is one which fails to realise all of these goods together, or one which fails to realise any, or one which fails to realise a proportion, or whether there is a mean on all counts which divides good from bad halves of a continuum, or whether some of these goods are more important than the others, or how I can tell if the list is exhaustive or not, nor, in short, do I have the first idea about how I ought to use such a list to assess a particular life for badness and compare human lots. Now of course, I do have some ideas about which of my friends and acquaintances are or are not having good lives just now, but as to consistency or unifying principles behind such thoughts, or as to how I ought to accord TH's 'Principle of Humanity' with these judgements, well I'm presently clueless.

More to the point TH's discussion leaves me clueless. Perhaps I can quantify 'a decent length of conscious life ' (I say: three score years and ten, though others nearer that age may demur). And Perhaps I can also, most of the time, competently tell people in agony from those that are not. But on the other desiderata I am at a loss to tell whether I or anyone I know has achieved them satisfactorily or not — or, for that matter, what would be satisfactory. Does Bill have freedom and power in various settings? About Mary, does she have satisfactory respect and self respect? James, does he get enough of the goods of culture, or does he work too hard? I just don't know how to apply these desiderata to an assessment of the lives lead by my friends and acquaintances, or to an assessment of my behaviour towards them. The 'Principle of Humanity' doesn't help us here in quite the way we would expect a moral principle — or a principle for assessing rightness in action — to help.

I imagine that, if pressed on this difficulty, TH might well admit that yes, his 'Principle of Humanity' (at least as it is presently developed) does not, as yet, help very much with telling right from wrong in our treatment of the human beings we westerners usually know personally, because in the usual cases we don't know exactly which of them are or are tending towards having bad lives according to these rather sketchy criteria, and know even less which actions would push their lives in which direction. TH's retort would be, I think, that his 'Principle of Humanity' was invented to help us tell right from wrong not in little matters, as it were, but on the large scale, between peoples and states and so on — and in these cases (he might say) statistics on life expectancy and medical emergency can be used to clearly tell the difference between peoples or states where most individuals are having bad lives, and other peoples or states where most individuals are not having bad lives, such that our assessment of right and wrong actions towards peoples and states can then be deduced from such accessible data. Refocused specifically on international politics, TH's 'Principle of Humanity' might possibly survive the objection that calculation of the various imponderables listed under the heading of the foreseeable consequences for promoting good lives is a daft way of assessing whether David punching his little sister was a case of right action.

The problem with this defensive strategy, from TH's point of view, is that it would raise the rather difficult question of why we should assess right action towards states and peoples on entirely different kinds of criteria to those on which we ordinarily assess right and wrong. It was precisely to avoid such a difficult question that TH claimed a universal range for his 'Principle of Humanity', presenting it as the correct method for assessing right action, period. This universal range is something it clearly does not have.

What TH offers us, inadequate as a definition of the bad life , is actually pretty good when considered as a sketch of the good life . Thus, in the good life I would have a decent length of conscious life , a bodily quality of life including absence of pain, freedom and power in various settings, respect and self respect, and the goods of culture, including knowledge, religion and more. Excellent. The mention of a bodily absence of pain is particularly redolent of nirvana. But what TH here claims to be offering is a 'clearly defined' picture of a bad life , and he gives us not a clue about exactly what connection is intended between some intended and 'clearly defined' picture of a bad life , and the picture he in fact offers us of the good life . I suppose we might say that any life falling short of the listed desiderata is not the good life , and is therefore is a bad life . That way of understanding the implied connection between the notion of the good life that TH gives us and the morally decisive notion of the bad life wouldn't suit TH, since it would suggest that, given the empirical evidence as to human perfection, all actions whatsoever have as their foreseeable consequences countless bad lives. From which it would follow that all actions are equal in their badness.

That's clearly not what TH has in mind. He means this moral calculus as a way of identifying the side of the just. He must have it that that there are some actions that lead more bad lives and some that lead to fewer, as if the edges of the bad life idea were evident and present in the world. But in what we have from him, TH's idea of a bad life is not delineated at all. He draws us a brief sketch of the good life and quits the scene, as if that alone were enough to furnish us with that 'clearly defined' picture of the bad life from which all right and wrong in action may derive. Rhetorically, TH leaves us with the impression that he has a worked out decision procedure for distinguishing right from wrong action — which procedure he can apply to Palestine. He in fact offers no such procedure.

Leaving the matter of 'Bad lives', note now that TH offers an interesting moral philosophy that I want to call quasi-consequentialism, where what matters in assessing the moral character of of an act is neither the consequences of the act, nor specifically that assessment of consequences forming the intentions of the actor, but a third thing: the consequences of that act according to 'the best judgement and information'. I spoke about these as the foreseeable consequences . The important point here is that the foreseeable consequences are not necessarily the same as the consequences foreseen or intended by the actor, nor always the same as the actual consequences . To bring out the way this quasi-consequentialist view departs from more familiar standpoints, note the way it permits TH to endorse the terrorism of both the Zionist Stern Gang and the radically islamist Hamas, despite the fact that many of the key intentions of both these groups are repugnant to TH, and despite the fact that TH also deplores the ultimate consequences of Zionism. This is possible because what he is endorsing here is neither the intentions, nor the actual consequences , but the foreseeable consequences of the two contrary terrorist campaigns.

Thus according to TH whatever the intentions or actual consequences of 1948 Zionist struggle, the foreseeable consequences according to the 'best judgement and information' were good. TH claims that the 'best judgement and information' supported the Zionist campaign as most likely to lead to preventing bad lives. Now, fifty years later, with newly available information about the bad lives of the palestinians, and on TH's assumption that Palestinian terrorism 'will succeed in the end' in creating a state where fewer people lead bad lives, TH is able to claim that the foreseeable consequences of Hamas suicide bombings make these terrorist actions right as well, right in just the same foreseeable consequences way that the Stern Gang bombings were right. For TH there need be no contradiction in having given moral justification to both the Stern Gang and Hamas, given that the morally relevant facts are the foreseeable consequences , rather than any facts about the actual or intended consequences .

Questions might be asked about the key details of this curious quasi-consequentialism, perhaps centrally about the meaning of 'best judgement and information', about who would be in a position to say what this is, and about what criteria for the identification of the 'best judgement and information' may be in operation. Obviously it would be good if the criteria here were more substantial than, say, whatever endorses the view I happen to hold at a particular time. TH does not address this point about criteria directly, but it is clear that he does not mean by 'best' something divine or ideal. Best human judgement and information can be fallible and incomplete, as for instance he holds to have been the case with the best judgement and information about the consequences of the creation of Israel. TH says nothing very helpful about what identifies the best judgement and information as the best, and this ought to be a little worrying. The Palestinians might have cause to object to TH that they were in a position to foresee consequences for bad lives, based upon their best judgement and information, that made the creation of the state of Israel wrong in 1948. The moral rightness of the creation of Israel in 1948 is something that TH endorses, but he does so on the basis of best judgement and information available to a non-palestinian perspective. Thus he argues, intriguingly, that the inconceivability to American eyes of establishing a Jewish Homeland in Germany in 1948 is an essential part of the rightness — which endorses — of the creation of that homeland in Palestine.

These are tricky knots. He implies here that there is a sense in which the contemporary American perspective on the postwar struggle for a Jewish homeland is nearer to 'best judgement and information' than is the information and judgement of the palestinian arabs. This is so despite the fact that the judgement of the palestinian arabs is the one he ends up endorsing as true, fifty years late, as they would see it. So patently, by 'best' in 'best judgement and information' he not only doesn't mean ideal, he doesn't mean truest either. But then what does he mean? What is it that privileges the US assessment in 1948, and the PLO or Hamas assessment in 2007? It is not at all clear on what basis TH would have us assess what is the 'best judgement and information', either today or at any given point in history. He makes no suggestions in his paper as to how we should identify this crucial part of his moral calculus. He simply asserts that this or that is, or was, the best available information and judgement. This is, at the least, something of an oversight.

To make one further point which is more important than the few words needed to state it would suggest, I doubt that TH is being wholly helpful in speaking of his morally relevant bad consequences as 'bad lives'. Hitler's life was, I would say, bad, given that it encompassed a fair degree of suffering and failure and ended in suicide. It was certainly not an enviable life . Likewise the lives of slave labourers in Wolfsburg were bad, in that their fates were unenviable. But I find it hard to imagine, as TH's picture would have it, that the badness in Hitler's life is morally relevant to antecedent actions in just the way that the slave labourer's was. I cannot have Hitler's sufferings enter into the same calculus with these other sufferings. This is, of course, to restate an old kind of objection to consequentialism, and one which I treat as decisive. But it is an objection which TH's innovation of quasi-consequentialism does nothing to escape. Instead of adding the necessary discrimination between morally relevant suffering and morally irrelevant suffering to the tradition of consequentialist moral theory, TH introduces two notions whose content he makes no satisfactory effort to precisify. 'Bad lives' is one such. 'Best judgement and information' is the other. For Honderich, what matters for rightness is the best judgement and information about the consequences for bad lives, notwithstanding that he says nothing to help us identify best judgement and information or to delineate bad lives from satisfactory ones in a way applicable to ordinary moral judgements, and notwithstanding, also, that some people rather deserve bad lives.

This point connects rather with what I said to Ricardo in another answer.

David Robjant

back

(23) Bryan asked:

Hello. I noticed that sometimes invalid argument forms can support valid arguments. For example, the syllogism:

1. No A are B.

2. All B are C.

.: 3. No A are C.

Is an invalid argument form, but can support a valid argument if A is 'cats', B is 'collies', and C is 'dogs'. Yet, I think there is something more intuitive that I am missing. My question is that does this invalid argument form warrant the valid argument above or is there a warrant that makes the 'cats collies dogs' argument still invalid? I would have thought that it would still be invalid because a cat could be a dog, but I thought that to be a poor reason since by definition a cat is not a dog.

---

OK, so you're right that it's an invalid argument form. If we accept that by definition a cat is not a dog, though, as you say the specific argument where A=cats, B=collies and C=dogs is valid. But that's only because the conclusion is necessarily true, because true by definition. So it doesn't matter what the premises are; any argument whose conclusion is 'no cats are dogs' is going to turn out as valid (semantic validity = it can't be that the premises are true and the conclusion false. So any argument with a necessarily-true conclusion is valid — so it's not surprising this one is! The argument 'All swans are white, so no cats are dogs' is also semantically valid, even though it is an instance of the manifestly invalid argument form 'All B are D, therefore no A are C'. There is no sense (in your example or this one) in which the argument form is warranting the inference, because it's the necessary truth of the conclusion that guarantees the validity of the argument, not anything to do with the internal structure of the argument.

I'm not sure how one does this with syllogisms, but in standard predicate logic one could think of the argument as having an implicit premise, (Ax)( if Cat(x) then not-Dog(x)). So then you'd get as your argument form:

1. -(Ex)(Ax & Bx)
2. (Ax)(if Bx then Cx)
3. (Ax)( if Ax then not-Cx) [implicit premise] So
4. -(Ex)(Ax & Cx).

This is a valid argument form, though a daft one because you don't need the first two premises. Oh, I should say, I'm not a logician! Probably there is a more technically sophisticated way of saying all this. But I'm pretty sure I'm right!

Helen Beebee

The British Philosophical Association
http://www.bpa.ac.uk

back

(24) Dave asked:

Is it possible to become significantly more intelligent in an hour?

---

Yes, there is some scientific evidence that this can be done. Listen to the music of Mozart for an hour (I think Bach might work as well). After this you will be more intelligent. I don't know how long the effect lasts but it will enable you to achieve a higher score on IQ tests.

Shaun Williamson

back

(25) Amarnath reddy asked:

Sir I always keep thinking about the past the days I spent whether they are good or bad memories and I keep on revising even minute memories I was never interested in the present or the future I always go back to the past days and keep trying to alter them but it is of no use I know

---

This could be a problem for you but only you can decide how much of a problem it is. If your obsession with the past has a bad effect on your life (i.e. it stops you doing the things you want to do) then you will have to get some help or counselling. Any obsession can be a way of avoiding thinking about your real problems. You can start to help yourself by not just thinking about the past, write it down in detail, this will enable you to distance yourself from it.

Shaun Williamson

back

(26) Ricardo asked:

I learnt about utilitarianism a while ago, through the work of Peter Singer, and I found it to be a very good ethical system. However, I reckon there are some vulnerabilities with this system. I heard one in a speech by Tom Regan, in which he asked if we should take into consideration the pleasure of a rapist before declaring rape wrong. I quickly came to the conclusion that an ethical system which only aims to minimize suffering would not be vulnerable to this argument and I found that there is indeed a system that requires us to promote the least amount of evil or harm, or to prevent the greatest amount of suffering for the greatest number, called Negative Utilitarianism. However, I'm facing a new challenge to this system after reading about a debate between Peter Singer and Richard Posner, in which Posner argues that if a dog threatened a human infant, and if it required causing more pain to the dog to get it to stop than the dog would have caused to the infant, then we, as human beings, favour the child. My moral intuition indeed tells me I would favour the child. Wouldn't I be incurring speciesism? Is there any way to justify my favouring the child without incurring speciesism?

---

The first question that comes to mind is — from what ethical principle(s) are you judging that it is inherently wrong to incur speciesism? Certainly, neither Mill nor Bentham had any notion of applying the principles of Utilitarianism to non-human entities. If you are going to attempt to include non-human entities within the scope of 'moral worth', then I think you need to provide a rationale. Or, at the very least, state your unsupported premise right up front.

I think you will find as much disagreement over the extent of what ought to be covered by the term 'moral agent,' or be granted consideration as due 'moral worth,' as there is over the particular ethical principles that ought to apply to moral decisions.

The second question that comes to mind is — what happens if we consider the scenario with a human pedophile in the place of the dog. If a pedophile threatened a human infant, and if it required causing more pain to the pedophile to get him to stop than the pedophile would have caused to the infant, then we, as 'normal' human beings, favour the child. I don't know about you, but my moral intuition certainly tells me that I, at least, would favour the child. And in doing so, I wouldn't I be incurring speciesism. But I would be violating the strictures of Negative Utilitarianism.

I would argue that such counter-intuitive scenarios provide as much negative press for Negative Utilitarianism, as similar scenarios provide for Positive Utilitarianism. I would suggest to you that if one's moral premise is any version of Simple Utilitarianism (meaning — other than Rule Utilitarianism), it is quite easy to generate relatively simple scenarios where the theoretical result is deemed by most to be counter intuitive. In other words, I think that on the basis of either form of Simple Utilitarianism, it is impossible to justify our intuitive favouring of the child. (Hence, the attraction of Rule Utilitarianism. Although that too has its difficulties.)

Which brings me to the third question that comes to mind — is an ethical system that generates counter-intuitive 'oughts' an acceptable ethical system. Or is there necessarily something wrong with such a result?

If you argue that any system that produces the occasional ethical recommendation that is generally considered contrary to our moral intuition, is an acceptable basis from which to guide our moral behaviour, then you must acknowledge that our intuitive favouring of the child is an immoral emotional bias on our part. A hold over (perhaps) from past cultural speciesism, that should be stamped out. Just the way that racist, sexist, ageist, (etc.) emotional biases ought to be stamped out.

But if you argue that our collective moral intuitions (in favour of the child in this scenario) are more acceptable guides to proper moral behaviour than the recommendations of some proposed ethical system, then you must acknowledge that Simple Utilitarianism (of either Positive or Negative flavour) is fatally flawed. And you must seek an alternative system of ethics that is more in tune with our collective moral intuition that places greater moral worth in the welfare of the child than in the welfare of either dog or pedophile.

If you favour the first approach, then you are going to have to argue your case against all those who would disagree with you — that some evaluations of our moral intuitions are erroneous. If you favour the second approach, then you are going to have to argue your case against all those who would disagree with you — with regards to whatever alternative system of ethics attracts your attention.

In other words, no matter what you do, you are going to have to argue your case against people who disagree with you.

(But if you do happen to choose the second alternative, and go looking for an ethical system more in tune with our collective ethical intuitions, then may I suggest that you take a look at Evolutionary Ethics.)

Stuart Burns

back

(27) Ricardo asked:

Hi. I learnt about utilitarianism a while ago, through the work of Peter Singer, and I found it to be a very good ethical system. However, I reckon there are some vulnerabilities with this system. I heard one in a speech by Tom Regan, in which he asked if we should take into consideration the pleasure of a rapist before declaring rape wrong. I quickly came to the conclusion that an ethical system which only aims to minimize suffering would not be vulnerable to this argument and I found that there is indeed a system that requires us to promote the least amount of evil or harm, or to prevent the greatest amount of suffering for the greatest number, called Negative Utilitarianism. However, I'm facing a new challenge to this system after reading about a debate between Peter Singer and Richard Posner, in which Posner argues that if a dog threatened a human infant, and if it required causing more pain to the dog to get it to stop than the dog would have caused to the infant, then we, as human beings, favour the child. My moral intuition indeed tells me I would favour the child. Wouldn't I be incurring in speciesism? Is there any way to justify my favouring the child without incurring in speciesism?

---

Posner slightly shot himself in the foot, by letting an unnecessary cross-species context into the point, which is a point against utilitarianism, positive or negative. So let's try to put that point better, and make species irrelevant. For 'dog' read 'romans' and for 'child' read 'christian'.

The romans are persecuting a christian in the arena. It happens that the several thousand romans would in total suffer more, for being deprived of their sport, than the christian does for undergoing the persecution (Lets be conventional and picture Lions).

Because there are enough persecuting romans that would each suffer if they could not to watch the persecution, the negative utilitarian calculation is decisively in favour of torturing christians for entertainment.

But it is wrong to torture for entertainment.

Therefore negative utilitarianism is a wrong theory of ethics (we've already agreed that positive utilitarianism is a wrong theory of ethics).

The point is: if the alleviation of suffering in those entertained by torture is to enter in to an assessment of the moral worth of that torture at all, it ought to enter in as a further point against doing it, and not for doing it, as the negative utilitarian moral proposal requires.

QED.

But one quick postscript on buddhists: I think they would say that the cause of suffering is illusion, and that the sadistic entertainment provided by the Romans (or by Channel 4) is actually deepening illusion and therefore (in the long term) causing suffering in the viewers, rather than as it may immediately appear alleviating it.

Possibly, therefore, the only way to make negative utilitarianism morally respectable according to our existing intuitions is: go buddhist. However, I wonder if intellectual attachment to an ethical theory of the sort you entertain is an adequate and solid ground for embracing buddha. The option is: reject utilitarianism.

David Robjant

back

(28) Scott asked:

Who is more intelligent?

A. A person who knows everything. is A (impossible?) Doesn't it depend on your environment? Student asked this question, I don't have a response for this HELP!! Right and wrong?

B. A person who knows he doesn't know everything.

I am teaching world history and my students attained this question from a Socrates quote: "I am the wisest man in the world, for I know one thing, and that is I know Nothing."

What is your opinion? and Do you mind if I share it with my students?

---

Socrates doesn't just say, 'I know that I don't know everything.' He says, 'I know that I know nothing.' And that should make us pause, don't you think?

It goes without saying that it is impossible to know literally everything. No-one knows, or arguably ever could know regardless of how long they spent investigating, how many grains of sand there are on all the beaches in the world, or how many planets in the universe have intelligent life. On the other hand, you can know everything about a sufficiently restricted subject matter: for example, you might be a British soccer fan who knows the names of every winner of the FA Cup since 1872.

It's good to be modest in the evaluation of one's knowledge. It may not be the whole of wisdom but undoubtedly it is an essential part of becoming wise to recognize our all-too human failings and the ease with which we gather misinformation without even realizing it. Any good teacher recognizes that they are not infallible, and that it is far better to admit to your students that you don't know than pretend to knowledge which you do not have.

On the other hand, if you ask me what I think of President Obama, and I reply that I have no knowledge of politics and never listen to the news so I can't answer you, you are hardly likely to be impressed by my great 'wisdom'. If one is aware of important gaps in one's knowledge, one should do one's best to remedy them, and not rest content with being an ignoramus.

But this is largely irrelevant, so far as Socrates is concerned.

Consider the following argument: If Socrates knows nothing then it follows (e.g.) that he doesn't know whether or not he's wearing his toga. For all he knows (namely, nothing) he might be out in the market place stark naked. If he gets arrested for indecent exposure, the judge and jury are unlikely to be sympathetic to his plea that he was unaware he was committing any offence. — How do you think Socrates would reply to that argument?

'My dear fellow, you must realize there is no knowledge of the things of this world. All we have are more or less useful beliefs. I believe that I am wearing my toga and am confident in this belief. For practical purposes, confident belief is all we need. Knowledge, supposing that any person had it, can only be of ultimate things, the answers to the deep questions of philosophy. Nobody knows that, although many think they do.'

This is rather poignant for me, because I have to ask myself what I have been doing for the last 37 years; what decades of pondering the great questions has achieved since I first started along the road to philosophy. It is a charge not infrequently laid at philosophers, that they never make any real progress, and the best they can offer is endless disagreement and scepticism.

But there is another side of the coin. Philosophers see things for what they are. They are not easily fooled by bullshit and propaganda. They understand what is important, and what are merely the trivial concerns of passing fad and fashion. The world (just as Plato said in his Republic) is not what you think it is. We are all cave dwellers. Even if Plato was over-optimistic in thinking that the philosopher alone has the power to escape from the cave, at least one can make the attempt — over and over again if necessary.

Geoffrey Klempner

back

(29) Yolanda asked:

Does the number 1 exist?

---

Strange question, like asking if blue exists. There are some things that are blue and there are some thing that are one, as in 'I have one television set'. Of course there are some things that are two as in 'I have two television sets'.

The word 'one' is an English word and it is a useful English word and that is all you need to know. Its existence will never be tested in a court of law. How could we ever have a reason to give up a useful word. Suppose we decided that one did not exist, then we would have to exclude from all our conversations. Would that be a good idea?

Shaun Williamson

back

(30) Ricardo asked:

'I learnt about utilitarianism a while ago, through the work of Peter Singer, and I found it to be a very good ethical system. However, I reckon there are some vulnerabilities with this system. I heard one in a speech by Tom Regan, in which he asked if we should take into consideration the pleasure of a rapist before declaring rape wrong. I quickly came to the conclusion that an ethical system which only aims to minimize suffering would not be vulnerable to this argument and I found that there is indeed a system that requires us to promote the least amount of evil or harm, or to prevent the greatest amount of suffering for the greatest number, called Negative Utilitarianism. However, I'm facing a new challenge to this system after reading about a debate between Peter Singer and Richard Posner, in which Posner argues that if a dog threatened a human infant, and if it required causing more pain to the dog to get it to stop than the dog would have caused to the infant, then we, as human beings, favour the child. My moral intuition indeed tells me I would favour the child. Wouldn't I be incurring speciesism? Is there any way to justify my favouring the child without incurring speciesism? '

---

Ricardo your question is a complex one, I will try to answer it as best I can. First let me introduce you to a form of argument called 'an elenchus' which was a favourite type of argument used by the philosopher Socrates.

Socrates would ask one of his students to define something like justice. He would them take take their definition and show how it would in many circumstances lead to very unjust actions. Socrates himself never offered a definition of justice and he may have had good reasons for that.

Utilitarianism is often presented as a definition of what is right or wrong (something I don't think it was ever intended to be) and we are then asked to criticise it in terms of does it lead to good or bad conclusions. This of course implies that we have an idea of what is good and what is bad independent of any theory of ethics. 'Surely Utilitarianism can't be right', we want to say, 'because it implies that we must take the pleasure of rapists seriously'.

When we do this we are applying an elenchus to Utilitarianism. However you then introduce the idea of speciesism into the question. Speciesism seems to be the idea that it is wrong to favour one species over another, a nonsensical idea that will lead to the extinction of many species.

If a dog attacks a child and I must not favour the child over the dog then when a tiger attacks a man I must not favour a man over a tiger. Then when a man attacks a tiger with a machine gun I must not favour the tiger over the man. Soon we will have no tigers and if we allow children to attack dogs, soon we will have no dogs. This whole idea of a moral principle that forbids speciesism, when you apply an elenchus to it, is nonsense. It will not protect animals, it will harm them.

Socrates knew something about our desire for neat definitions and how it could lead us to nonsensical conclusions. The wish for a neat definition of right and wrong is a useless quest. However this endless search for neat definitions, the idea that we can reach knowledge by coming up with just the right definition continues to mesmerise philosophers.

Shaun Williamson

back

(31) Tiffany asked:

If it is true that electrons blip out of existence for a short while and than reappear I was thinking that in a theoretical 'parallel universe' might be sharing our electrons. if that could be true than what happens when you die and the energy you use to move, think, have emotions etc. blips out of existence. I was thinking that you start over. energy cannot be destroyed it can only change form. If that is true than heaven is now and always. It would make sense. you would eventually be everything. you would be a boy, girl, president, a warrior and a coward and it would last an eternity because human imagination is eternal. I just thought it would be cool to have a great thinker ponder my idea and give a replay, thank you.

---

Do not confuse the conservation of energy with the persistence of identity. Just because the energy of your body is not destroyed at your death does not mean your identity is not destroyed. Imagine that a raindrop, as it falls, evaporates until it is all gone; does its identity continue to exist? Also, what do you mean by saying that human imagination is eternal? Do you mean limitless, infinite? That would not require time to be infinite, which is what is usually meant by eternity. By the way, I am no great thinker — just a trained thinker.

Helier Robinson


back

(32) Yolanda asked:

does the number 1 exist?

---

Platonists say that it does exist, and formalists say that it doesn't. It is said of mathematicians that they are all formalists on weekdays and Platonists on weekends. In other words, you have to make up your own mind on this: the answer is unclear.

Helier Robinson


back

(33) John asked:

In evolution, it generally makes sense that an inhibitory function evolves AFTER the thing it is inhibiting.

As an example, the human has layers of cortex devoted to inhibitory behaviour, which effectively limit the lower brain layers.

The basic fight and flight instincts of the older brain, in addition to sexual drives, etc. are inhibited to stop us running around and impulsively just killing/raping/Stealing, etc. The inhibition layer comes after the earlier layers; meaning that prior to the inhibition we did actually just run round killing, raping, stealing impulsively (like on a Friday night out in Barnsley)

Now the interesting thing is that there are some inhibitory brain functions devoted to limiting the control that we can consciously exert over our own minds (mental self control).

This makes perfect sense from a genetic perspective if an individual can 'think' away hunger, they will be less inclined to find food, if they can 'think away' pain, they will be less likely to avoid noxious/aversive stimuli. If they can think pleasurable thoughts, which were as intense as the real thing, then the drive to achieve pleasure, e.g. through sex, becomes redundant, again so the survival chance lowers.

Now as these mental self control inhibitory processes do exist (e.g. ironic processes of Wegner) it assumes that prior to their evolution, there was a brief period where for a short time there existed individuals with full mental control, i.e. they could think away pain, think away hunger, etc. These could have been individuals who could have been true 'Lotus Eaters', absorbed in their own conscious worlds, and distanced from real world controls such as pain, hunger, sex, etc.

Enlightened apes?

---

Lower and higher? Whoa there Mr teleologist!

In evolution, it generally makes sense that an inhibitory function evolves AFTER the thing it is inhibiting.

As an opening assertion, that's both attractive and baseless. It's an appeal to the beauty of a picture rather than to any point of fact. I mean, if the world were like this, it would be neat, wouldn't it? But there is really no adequate reason to think it is in fact like that, and you rather acknowledge as much with the use of the appealing and vague expression 'it generally makes sense that' in preference to an expression that would be more obviously falsifying. If you had said: 'it is necessary that an inhibitory function evolves AFTER the thing it is inhibiting' I could have answered simply by pointing out that no such evolutionary necessity exists. What you mean is that it would appeal to your sense of order and reason if an inhibitory function involved after the function it was to inhibit. But evolution is neither ordered nor rational, and you use terms that misrepresents what evolution is.

To explain.

Your argument rather ignores that selection acts upon random mutations, and that mutations do not pop-up to fulfill a discrete 'function'. Rather, mutations or DNA copying errors mess about with the coded creature in any number of ways that bleed across lots of different things that we are only much later able to call 'functions'.

Thus, there is no particular reason of any kind why some mutation can't lead perfectly well both to an antelope being more skittish re big cat noises, and to it's being more reflective about sexual partners.

Your argument also suffers from dividing 'functions', which as I have just pointed out are anyway not the things upon which evolution acts, into higher 'inhibitory' and lower non-'inhibitory' kinds. I'd like to know from what perspective we are to make this distinction? You seem to assume that whatever it is that enables a buddhist monk to sit reading improving texts must be something inhibitory, as if all that were required to behave like this was not to behave in some other way. But that's obviously false. There is no particular reason why I couldn't regard his ability with the scroll as the positive ability, and think of the drive to impregnate the nun in the next room as the inhibitory function. Except, of course, that this contradicts your claim that inhibitory functions are the ones that evolve last. But, as I have already pointed out, that claim is baseless on several levels. For one evolution doesn't act on functions, for another there is no one to one correspondence between a function and a mutation, and finally there is no privileged perspective from which we might see which functions are to count as 'inhibitory'.

There is no particular reason, just for instance, why the capacity to stand up should evolve after the capacity to fall down.

David Robjant

back

(34) Tristen asked:

How can humans prove that they exist?

---

Why should they wish to?

I realise that kooky ideas are fun, but what human being doubts that they exist? I mean, actually doubts it, as opposed to thrilling themselves with words and imaginary possibilities, as a cineaste thrills to dracula?

Supposedly, Rene Descartes managed to doubt his existence at least temporarily. Well, that's what he says. But why take his word for it? What, after all, is he able to offer us as evidence of this doubting?

Look, I can say 'I doubt that 2+2=4'. Wow. But what does my saying this actually show? In point of fact I have never wavered from treating 2+2 as 4. It is much the same with my existence (and with descartes', for that matter).

Proof has a function only where there is doubt. If, verbal dexterity not withstanding, I have not in fact managed to doubt my existence, it follows that I have no need of any proof.

David Robjant

back

(35) Ricardo asked:

I would like to comment on my previous question. From the answers I received, it seems most of you don't know the work of Peter Singer and that has lead you to some incorrect assumptions.

In one of the answers I received, Shaun Williamson says 'If a dog attacks a child and I must not favour the child over the dog then when a tiger attacks a man I must not favour a man over a tiger. Then when a man attacks a tiger with a machine gun I must not favour the tiger over the man.'.

This might be right if we consider speciesism to be 'the idea that it is wrong to favour one species over another'. Well, for Singer and for me as well, what is wrong is 'not attributing equal consideration of interests based merely on a difference os species'. Combine that definition and preference utilitarianism and you can not make the conclusion Shaun did. A man has a bigger interest in not getting killed than a tiger does therefore we wouldn't favour the tiger. Peter Singer makes this point very clear in his Animal Liberation book. I strongly suggest you become familiar with his work before discarding both utilitarianism and speciesism as 'nonsensical'. To seek knowledge is also part of being a philosopher, after all.

---

Well I do know the work of Peter Singer and I don't regard it as being very important. If a dog attacks a child and you stand there wondering if stopping the attack would make you guilty of speciesism, then no one is going to understand your attitude, no one is going to praise your cool intellectual consideration of the matter. Singer has no real idea of how much a tiger fears death, he is just making things up as he goes along. I strongly suggest that you get a sense of proportion before believing in nonsensical philosophical speculations. I do not know of one single person who bases their moral beliefs on utilitarianism. If you can do this then you will become the first utilitarian in the world. Good luck with that project.

If you are ever going to reach real knowledge then you need to be able to sense nonsense when you hear it, just as Socrates did.

Shaun Williamson

back

(36) Aup asked:

Here goes my question.

If you were trapped on the top of a mountain with only a giant slinky, a ferret and an exercise bike, how would you escape your dilemma?

Many thanks in advance for your reply.

---

What dilemma? I don't see any dilemma.

Shaun Williamson

back

(37) Andrew asked:

Can you point me in the right direction to answer the following. I exist. There is no way that I can logically see that 'I' could not exist. To believe otherwise would be to accept that there are entities that do not exist, which I do not see as possible. My question is then, does this mean that I had to exist? That is not a matter of incredible chance but a certainty?

---

It's not an uncommon feeling — the thought that somehow I had to exist, that there is no logical possibility that I could have failed to exist. For some, the train of thought does not end there. When I consider the prospect of my death, it seems impossible, for very similar reasons (if one can talk of reasons) that there will be a time when I do not exist.

But let's just concentrate on the first claim. In order to be here, writing this, my father had to produce the sperm that fertilized my mother's egg, which grew into a foetus and eventually became me. If the sperm and egg had not come together, I would not have existed. But exactly the same applies to the existence of my parents, and their grand parents, their great grand parents, and so on. If any one of those links had been broken — going right back to the beginning of the human race — I would not be here today. All in all, an incredible chance, a fantastical improbability.

It's almost impossible to believe. But let's just look at the alternative. I had to exist. I could not have failed to have been born. Am I willing to grant the same about you? Not at all. I have not the slightest difficulty in supposing that 'Andrew' (whoever he is) might not have existed, in which case I would have had to find some other question to answer today. (The supply of questions, and questioners, is never-ending.)

If I had to exist, but no-one else had to exist, then I must be very special. Maybe I'm God? That's a thought that might have occurred to one or two people. Fortunately, common sense and sanity usually prevail. It's generally considered acceptable to think of oneself as 'special', in the sense that one's relation to one's own existence has a unique character or flavour which is absent from one's relation to others. However, that hardly suffices to alleviate the sense of dizzying vertigo at the paradoxical improbability of one's own existence.

We are all in the same boat; that's true. The same logical problem applies to you as applies to me. I can only feel the paradox in my own case, just as you can only feel the paradox in your own case. But that's no help. I fully empathise with your saying what you say, because I'm motivated to say it too. The difference is that your saying what you say has an obvious explanation. That's why I'm not the least bit puzzled by the contingency of your existence. Why, then, can't I apply the same explanation to myself?

I can't. That's just a brute fact. That is what it is to be the possessor of a perspective on the world, a subjective standpoint. Yet, strangely, this observation does seem to point to a possible resolution. The sense of paradox doesn't go away. Rather, I get to see it for what it is: simply an inevitable consequence of the fact that I am stuck here, unable to step outside my own point of view even for a moment. There's something I can't see, not because there is any obstruction to my vision but because the very act of seeing places me here and not there.

I am not saying that we are unable to think about how things are from other points of view. It is built into the very nature of human language that we can contemplate what it would like to be in someone else's shoes, or even conceive of how things are from no particular point of view (how would you do science otherwise?). Yet, in all this, remains the stubborn fact — I am inclined to call it a stubborn metaphysical fact — that, necessarily, I am always the one asking the question. I can pose your question to myself; but I can't ask your question for you.

Geoffrey Klempner

back

(38) Esther asked:

How do helpful variations accumulate in a species over time?

---

The answer lies in the word helpful. A helpful variation, by definition, helps an animal to survive and breed.

By chance some elephants are born with a gene that leaves them without tusks. When poachers were killing elephants for their tusks, they ignored the elephants without tusks. The tuskless gene survived and became more common. In some ways not having tusks can make life difficult for an elephant since it is the main means of defence against enemies. However if it means that poachers won't shoot you then it can become an important survival mechanism.

Shaun Williamson

back

(39) John asked:

In evolution, it generally makes sense that an inhibitory function evolves AFTER the thing it is inhibiting.

As an example, the human has layers of cortex devoted to inhibitory behaviour, which effectively limit the lower brain layers. The basic fight and flight instincts of the older brain, in addition to sexual drives, etc. are inhibited to stop us running around and impulsively just killing/raping/Stealing, etc. The inhibition layer comes after the earlier layers; meaning that prior to the inhibition we did actually just run round killing, raping, stealing impulsively (like on a Friday night out in Barnsley)

Now the interesting thing is that there are some inhibitory brain functions devoted to limiting the control that we can consciously exert over our own minds (mental self control). This makes perfect sense from a genetic perspective if an individual can 'think' away hunger, they will be less inclined to find food, if they can 'think away' pain, they will be less likely to avoid noxious/aversive stimuli. If they can think pleasurable thoughts, which were as intense as the real thing, then the drive to achieve pleasure, e.g. through sex, becomes redundant, again so the survival chance lowers.

Now as these mental self control inhibitory processes do exist (e.g. ironic processes of Wegner) it assumes that prior to their evolution, there was a brief period where for a short time there existed individuals with full mental control, i.e. they could think away pain, think away hunger, etc. These could have been individuals who could have been true 'Lotus Eaters', absorbed in their own conscious worlds, and distanced from real world controls such as pain, hunger, sex, etc.

Enlightened apes?

---

Well you paint an interesting picture but I'm sure you know that is is just a picture and evolutionary biology and evolutionary neuroscience cannot be done just by thinking about things and adopting appealing theories.

The enlightened ape who can think away pain won't survive very long in a world where pain is a valuable warning of what to avoid.

A really wise ape would pay attention to those hunger pains. The idea of an individual with full mental control is the idea of the heroin addict or the alcoholic and they don't survive all that well. The brain is the greatest evolutionary survival mechanism that exists. Thinking it away is the greatest survival mistake that any animal could make.

Shaun Williamson

back

(40) Suzanne asked:

What is one difference between socialism and communism?

---

'Socialism' can mean anything you want it to, and there are those who by using it merely propose their own conception of the common good. There are others who by using it mean to slur such a conception of the common good and thereby avoid paying more tax.

The slur can be unpicked by examining a more precise meaning of 'socialism' in which the distinction between communism and socialism is tightly drawn — and then contrasting that idea of socialism with what Clement Attlee stood for.

For a Marxist-Leninist, Communism is the objective, and Socialism is the means. To put it another way: communism is what society looks like when it no longer needs government (the state 'withers away'), whereas socialism is what society looks like when it's being beaten and whipped towards communism.

To explain: the marxist hypothesis (owing something to Rousseau's romanticism) is that all human beings will be perfectly co-operative and virtuous towards each other, and thus will no longer need government, just as soon as the distorting effects of capital and private ownership are removed. That ideal utopia would be called communism. But capital and private ownership are difficult to eradicate (partly because of the class interests of the bourgeoisie, and partly because it is tricky to eradicate an idea), and so on the way to stateless communism a certain, ahem, 'temporary' intensification of the powers of the state will be necessary. In rousseau's expression, we may be 'forced to be free'.

You will have noticed, alas, that the soviet mass graves and gulags did not pave the way to a stateless utopia of all the virtues. Of course, you might account for this historically — any keeper of the true faith is likely to argue that communism hasn't been arrived at yet because the socialist stage was just not done properly. The most global form of this argument is the one that says that Russia was 'not ready' for socialism, because it had not passed through the stages of social organisation in the order proscribed by Marx (after observing Manchester and supposing it a model of the world). Note, by the by, that Marx didn't actually proscribe anything of the sort: he claimed, rather differently, that such an ordered progression of socio-economic systems is 'inevitable', to which claim the case of soviet Russia is, ironically, an obvious counter-example. But the argument that communism was not achieved because socialism was not done properly is dim for other reasons.

The parallels between the abrahamic religious story of the Fall and the Marxist one have been widely noted (rousseau is an intermediary). On both accounts there is one reason, one fact, that accounts for all the moral horrors, all sin, all vice, cruelty, injustice. On the one account, that eve ate the apple of the tree of knowledge of good and evil, on the other, if you like, that she took ownership that tree (means of production). On both accounts: but for this one mistake all in the garden would be perfect.

Now, the better sort of religious person takes the story of the fall as an instructive moral myth, which it might take daily devotion and a life of study to gain and practice the full meaning of. The meaning of the story is certainly not that we must watch out in case a serpent offers us an apple. Marxism, however, is a refuge for the literalist: there was literally one founding mistake, like eve taking the devil's gift, and whereas eve cannot be got to un-eat the apple, perhaps her daughters can feasibly be got to un-own the tree. Then the peace and order of the garden will return — all trees and apples being held in common: communism.

Don't you want to return to the garden? A very large number of people do. I particularly hoped for it in early adolescence.

Then it occurred to me, or rather I learned, that there might be lots of reasons why people were mean to each other besides ownership of the means of production. I rejected the idea that one need merely undo some original sin.

Rather as in certain kinds of religious zealotry, the fantastic otherworldliness of the communist utopia implies a dangerous contempt for the world as it is, and the people in it — people who precisely because they are real and complex appear to obstruct the path towards the beloved simplicity of the fantasy. Humanity needs to be made anew in the image of the viceless future, it is argued by the communist (much as by the zealot). What this means in practice is all the people you disapprove of may be murdered in the name of virtue — a licence for horror without end. You see the faith is that the utopian delights will arrive, but need not do so just yet.

Thus in the ideology of the old left:

Communism is the great 'not yet', the kingdom of heaven on earth.

Socialism is the necessary horror, a means justified by the end.

I leave out the finer post-leninist points, such as the diverting (and deadly) controversy over 'socialism in one country'.

I think it more worth saying that Clement Attlee, for instance, had none of the other-worldly ambitions implied by the leninist picture of socialism as a means to communism. He never pictured the state withering away, or justified current intervention in terms of that withering. He never imagined some reversal of the Fall. He was too english, insufficiently totalitarian in outlook, to plot the overcoming of any original sin. He just wanted people to have jobs and roofs over their heads. Attlee's Labour Party would not have qualified as Socialist under any leninist criteria, thank god. Thus the idea that the contrast between Lloyd George's liberals and Attlee's Labour is an ideological one is deeply mistaken.

David Robjant

back

(41) Robert asked:

I have a couple questions relating ot religion. One of the philosophers in 44/52 said that theology was not philosophy. There is also a philosophy of religion though, and many philosophers who have been profoundly religious, and they let that fact shape their philosophy. Can someone please elaborate then on the difference between the philosophy of religion and theology?

Also, I have had a hard time trying to come up with a definition for religion. Is religion separate from spirituality? If so how? Is it simply in the following of a dogma that something becomes religious?

---

Well I will try. Philosophy is concerned with truth not with belief. So philosophers deal with questions such as can we prove that God exists.

Theology deals with the beliefs of a particular religion. So a Christian theologian believes that Christianity is true and explores the consequences of this belief. The philosopher of religion as a philosopher does not advance the truth of any particular religion.

So for example 'Does the Bible teach us that homosexuality is evil?' or 'Does the Koran teach us that that women are inferior to men?' are both theological questions and are of no interest to philosophers of religion.

Of course sometimes the two subjects collide. The fundamentalist theologian may hold that God created the world in seven days 4700 years ago. The philosophers may ask is this belief reasonable or is it just nonsense. However in this case the philosopher is not concerned with what the Bible teaches, he is just concerned with is it reasonable to consider such a belief as being true.

I wouldn't worry about not having a definition of religion there are many things that don't have neat definitions, for example we all know what a game is but as Wittgenstein pointed out there is no neat definition of what a game is.

Shaun Williamson

back

(42) Angie asked:

Which ethical theory is most natural? That is, which theory requires the least alteration of our natural inclinations in order for us to live ethically?

---

Why do we need a ethical theory. What is it going to do for us? Suppose we adopt theory X and then in some circumstances this theory requires us to perform an action that seems to us to be clearly wrong, then we will say that the ethical theory is no good and we need a better one.

There is no neat definition of right and wrong. It is a philosophical illusion to assume that all complex abstract ideas can be summed up by some neat theory.
We assess ethical theories according to whether they seem to lead to good or bad conclusions, so we have an idea of good and bad that is prior to and transcends any ethical theory.

Shaun Williamson

back

(43) Nav asked:

What is Plato's Theory of Forms? What problems in metaphysics, epistemology, and ethics is it intended to solve? Does it work?

---

Plato's Theory of Forms is the view that, as opposed to the world of ordinary experience, which takes place in our physical world, there exists a world of real, stable, permanent, and immaterial Forms, Forms which are in some sense the cause of the objects of our ordinary experience. Plato intended the Forms to solve problems in epistemology in the following way. Our everyday statements include the use of general terms (like say, 'cat'). In order for our ordinary statements to be meaningful, we must know what these general terms signify or refer to. But to do this, Plato thought, we must do more than point to particular things. So, for instance, if we want to know what the general term 'cat' refers to, it will not do to point to 'Tom' in the Tom and Jerry cartoon, or to the stray cat down the street. These are merely particular examples or instances; they do not tell us what the general term 'cat' refers to, or what its meaning is. But all our sense experience gives us are examples, illustrations, or instances of general terms. So how can we ever come to know the definition of 'cat' or what it refers to while we are here, in the everyday, corporeal world? Plato believed that we all have knowledge of the Forms (for instance, the Form of Cat) within us (our disembodied souls were once acquainted with them while between physical lives), however, this knowledge is latent. What our sense experience does (seeing Jerry the cat and the cat down the street) is jar our memory of the Form of Cat, bringing to our conscious awareness information that is deep within us, of which we were not aware of until now.

The same goes for ethical terms. In Plato's Euthyphro, Socrates meets Euthyphro, a man who claims with certainty that persecuting his father for murder is the 'holy' thing to do. Socrates asks him what holiness is. Euthyphro responds that it is precisely what he is doing, going to court to have his father tried for murder. But this act, Socrates responds, is merely an instance or illustration of holiness, not the meaning of the general term 'holiness'. Euthyphro proves unable to supply this meaning. But how then, Socrates goes on to ask, do you know that this act you are about to commit is holy, if you do not know what the meaning of 'holiness' is? Again, Plato's Forms are intended to solve this difficulty. If we have access to (through recollection) the Form of Holiness, then we will be able to know what is required for some particular act to be holy. If we know this, then of course we will be able to tell whether or not Euthyphro's act is holy.

Does this work? I suppose that would depend first and foremost on whether or not one believed in the metaphysical claim that there is somewhere or other a Form or Ideal of Cat, immaterial, unique and unchanging, and with it, that our souls, in a non-corporeal state, become acquainted with these Forms. After all, we might simply wonder if we really need such an elaborate theory to help explain abstract ideas or general terms. Why might we not simply say that the general term 'cat' exists because we have had experience of cats and need a word to describe them. In this sense, general terms are not seen to exist eternally in a world of Forms, waiting to be discovered by the intellect alone, but are developed from our continual experience of physical items. It might be interesting to note that this is exactly what Aristotle, Plato's pupil, believed.

Kristian Urstad

back

(44) Klaus asked:

David Hume is quoted as 'the passions rule the mind' or 'reason is handmaid of passions'.

Just reviewed 'Enquiry Concerning Human Understanding' and cannot find such a quote nor does it fit his emphasis on experience and the reasoned interpretation thereof.

Could you clarify please.

---

It's 'Reason is, and ought to be the slave of the passions', and it's in the Treatise, Book II, Part III, section 3 ('Of the influencing motives of the will'). The other famous quote is ' 'Tis not contrary to reason to prefer the destruction of the whole world to the scratching of my finger', which occurs a couple of paragraphs further down.

Helen Beebee

The British Philosophical Association
http://www.bpa.ac.uk

back

(45) Ricardo asked:

I would like to comment on my previous question. From the answers I received, it seems most of you don't know the work of Peter Singer and that has lead you to some incorrect assumptions.

In one of the answers I received, Shaun Williamson says 'If a dog attacks a child and I must not favour the child over the dog then when a tiger attacks a man I must not favour a man over a tiger. Then when a man attacks a tiger with a machine gun I must not favour the tiger over the man.'.

This might be right if we consider speciesism to be 'the idea that it is wrong to favour one species over another'. Well, for Singer and for me as well, what is wrong is 'not attributing equal consideration of interests based merely on a difference os species'. Combine that definition and preference utilitarianism and you can not make the conclusion Shaun did. A man has a bigger interest in not getting killed than a tiger does therefore we wouldn't favour the tiger. Peter Singer makes this point very clear in his Animal Liberation book. I strongly suggest you become familiar with his work before discarding both utilitarianism and speciesism as 'nonsensical'. To seek knowledge is also part of being a philosopher, after all.

You say 'A man has a bigger interest in not getting killed than a tiger does.'

How could you possibly know?

---

I answered your question before, but your response to Shaun provokes me.

For what do 'interest' and 'preference' mean here? If they mean to refer to anything recognisable as an interest or preference, such as a will to go on living, then to put your proposal into practice we would have to devise some way of asking the tiger and assessing his love of life against that of the man — which looks rather tricky to arrange.

Or do 'interest' and 'preference' in fact serve as disguises for 'value'? You seem to present the thought that there is more value (interest) in a man's life compared to that of a tiger as wholly in accord with the indictment against unequal treatment on the basis of species. It patently is not.

Moreover, if 'interest' and 'preference' do mean 'will', then you recommend a wholly inappropriate method for assessing the moral character of killing. If I make someone's life so miserable that they want to die, and then kill them, is that OK? Is that torture followed by murder, as the account would imply, more OK than if I had treated the man with every kindness right up to the point where I murdered him, so that, at that point, he really loved life ? This moral view would be preposterous.

David Robjant

back

(46) Jake asked:

What is the purpose of aesthetics? Why are we so moved by poetry, music and art? And why do our tastes of such things differ from one person to the next? None of these are necessarily essential for survival, yet we hold them in high regard in our life. Furthermore, our tastes for such things change over time. Can this be explained in a logical sense? Is man a rational animal to hold these things in such high esteem? How are these things pertinent to us in our day-to-day individual fights for survival we call life?

---

I'm writing this to the accompaniment of Ministry of Sound's Clubbers Guide to Ibiza (2001). Usually I prefer silence when I work, but today I just felt the need for something with a beat. I've never clubbed and never holidayed in Ibiza, yet this music moves me. I can feel the beginnings of a smile twitching around the corners of my mouth. (Maybe the thought of bikini-clad beach goddesses has something to do with it too — but that's something we might talk about later.)

Jake's question about art is posed against the background of the human 'fight for survival' and the things that are 'essential for life'. It could be argued that he has stacked the deck right from the start. Why should we accept that the things we value must be measured against that austere standard?

What is the value of life? or survival? How many hours (or minutes?) of my life would I give up for the chance to listen to this great CD? Perhaps not that many, but still it's a fair question. Life is finite. We are all going to die some time. So it's legitimate to ask how far you theoretically value quality of life over quantity — if at all.

Jake's follow-up questions effectively answer his first question: 'What is the purpose of aesthetics?' The philosophy of art — or aesthetics — seeks to account for the value that the objects of art have for us. One of the challenges for aesthetics is that this value is not constant, either between different individuals, or for the same individual over time. To that extent, aesthetic value is fragile and evanescent.

One school of aesthetic thought that deserves to be reckoned with is that the objects of art have value for the pleasure they give us. What makes art different from other pleasures is that calling something 'art' implies a standard. Quality is important, not just quantity. It takes refined taste to discern the quality of pleasure.

To take the present example, I don't know a lot about dance music CDs but I know what I like. Glancing at the reviews for Clubbers Guide to Ibiza I can see that this two CD set is rated highly, although there were some criticisms. I couldn't write a review myself because I don't know enough about DJs and mixes and suchlike. My taste is not sufficiently 'refined'. But my ear is capable — if I strain to make the effort — of picking up on some of the points noted in the more critical reviews.

But hold on a minute. Isn't there a much more fundamental question that we still need to ask? Why does music — or a painting, or a novel, or any art object — give us pleasure? Surely it is a remarkable fact about human nature that we like certain combinations of sounds and dislike others; or that we enjoy looking at certain kinds of object; or that we are moved by the fate of characters in fiction.

Of the three remarkable facts, the pleasure of looking seems the least difficult to explain. To the question, 'Why is a man (woman) so moved by the sight of a beautiful woman (man)?', one can only answer, 'That's just the way human beings are.' So is man a rational animal to hold sex in such high esteem? A case could certainly be made that contemporary culture is over-obsessed by sex, but that is just a matter of relative values.

This approach to aesthetics seems to fly in the face of accounts (like the theory proposed by the gloomy Schopenhauer) which stress the 'disinterested' nature of the aesthetic response. However, I would accept that hedonism is least useful as an explanation of the intellectual content of the art work, the fact that the artist is not merely setting out to please but making a statement.

An example of what I mean would be Picasso's great painting Guernica which depicts the horrors of the Spanish Civil War. Picasso's painting is not meant to be pleasing, or easy to look at. Yet we value the immensely skilful means he has chosen to convey his message. We feel that we understand more about the tragedy of the bombing of Guernica in 1937 than we could have learned from news photographs, or a literal account. The painting gives shape and depth to our emotions. Surely, anything which gives shape and depth to human emotions enhances the quality of human life.

So my answer to Jake would be that if the quality of life matters at all, then art surely does matter.

Geoffrey Klempner

back

(47) Nav asked:

Dave's car's fuel gauge, which has always been reliable, says he will run out of gas soon. But Dave is convinced that there is no necessary connection between causes and effects, and no guarantee that the future will resemble the past. So Dave doesn't buy any gas. Dave runs out of gas in Gilroy and calls you to come and rescue him. Should you be annoyed at Dave for being irrational? To what philosopher(s) does Dave owe these views? How does that philosopher support these views? What philosopher(s) have opposed these views, and why?

---

It seems hardly worth answering this. Dave obviously does believe in cause and effect since he believes that picking up a telephone and dialling your number will connect him to your telephone. He also believes the future will resemble the past since he believes that if he asks you, you will come and rescue him.

Dave is an annoying, irritating git and I advise you to ignore him. Forget about philosophers who have said that there is no necessary connection between cause and effect. There is a connection between cause and effect, it isn't a necessary one and we don't need a necessary one.

Shaun Williamson

back

(48) Patrick asked:

Are Wittgenstein's views on language basically the same as the the Sapir-Whorf view that it is language moulds our thoughts?

On the other hand, which Philosophers (if any) support the opposing 'cloak' view, which I guess is Pinker's basic position in proposing the idea of 'mentalese'?

---

Well I suppose I will have to displease you by saying that in his later work Wittgenstein did not hold either of these views. It is not the aim of philosophy to arrive at any view of what language is, nor can it do any such thing.

Wittgenstein wrote 'Philosophy is a battle against the bewitchment of our own intelligence by means of our own language'. It is the task of philosophy to free us from the temptation to say things like 'language moulds our thoughts' or the temptation to propound absurd ideas like mentalese.

In doing philosophy we must become absolutely clear about the things that trouble us, then we will no longer be tempted to think that there are any conclusions to be reached.

Shaun Williamson

back

(49) Andrew asked:

Are philosophers getting better at philosophy. If so who is the best.

If not why bother?

Are any philosophers totally superseded. Which ones? Can you start with the best philosopher and ignore all the 2nd rate. If not why not.

Is philosophy the study of itself.

---

I will answer each of your questions in turn. Its all a matter of opinion if philosophers are getting better and who is the best. I think Wittgenstein was the best and nobody is any better.

Don't bother to study philosophy unless you feel compelled to to do so. If you study it because you think it might be interesting, cool or trendy then you will be disappointed. If you can avoid philosophy then do so.

No it is not the study of itself. That would be the Philosophy of Philosophy and I'm not sure that that exists. Perhaps you could invent it.

No, philosophers are never superseded, there is no progress in philosophy so no one ever becomes obsolete.

Shaun Williamson

back

(50) Andrew asked:

Questions are not only words are they? I can imagine a world where thoughts are constructed from all sorts of signs and symbols. I can imagine worlds where beings think with their hands or with their eyes or with a combination of both hands and eyes. I learnt to draw by questioning with my hands and eyes and never a word passed my brain. I am a excellent drawer now.

If some questioning is embodied we are going to struggle to put it into words aren't we? So can it be said that a

good question is only one that can be put into words?

Can we question embodied questions.

Embodied question.

Eyes and hands react to smudge of charcoal on paper and neither rub it out or add to it because the answer to the question was leave it alone. But non of this happened in words.

---

It is possible to confuse yourself by playing with words. Talking about questioning with your hands is just a metaphorical way of talking unless you are using sign language (which consists of words and sentences, just as any other language does) and of course in any language certain signs and gestures can have a meaning, like the signs in the highway code.

Both good and bad questions can be put into words but if there are any questions that cannot be put into words how are we supposed to answer them or even understand them?

So suppose I write 'He asked me a question that could not be put into words and I gave him the answer straight away'. Now if we allow that to be true then why not 'He asked me to solve a mathematics question that could not be expressed in words'. Maybe even 'I asked the waiter for the bill but he said that it could not be expressed in words'. Where will it all end?

Remember that a drawing requires an interpretation. A drawing of a man walking uphill could also be a drawing of a man sliding downhill backwards. However you should also remember that pointing is part of our language as are other gestures.

Philosophical questions always have to be expressed in words as do many other sorts of questions despite what you think you can imagine. Question is a word and questions are primarily expressed in words. Our primary idea of a question is that it is a sentence that asks something. A sentence is a linguistic unit.

Shaun Williamson

back

(51) Robert asked:

I have a couple questions relating to religion. One of the philosophers in 44/52 said that theology was not philosophy. There is also a philosophy of religion though, and many philosophers who have been profoundly religious, and they let that fact shape their philosophy. Can someone please elaborate then on the difference between the philosophy of religion and theology?

Also, I have had a hard time trying to come up with a definition for religion. Is religion separate from spirituality? If so how? Is it simply in the following of a dogma that something becomes religious?

Thanks for your time

---

As the respondent of 44/52 I will answer the first part by expanding on what I think the main difference between philosophy and theology is. Philosophy as it has developed since Plato and especially since Descartes concerns itself with general issues such as the nature of reasoning, (logic), the nature of knowledge (epistemology), the nature of being (metaphysics) the nature of language and how we can live together (Ethics) (See 43/67 for further details) . It tries to do this in as dispassionate manner as possible without the use of faith or claims to divine revelation from either a personal experience of God, or from a sacred text such as the Bible or the Koran. As I have mentioned before (43/81) the idea that the answer to an ethical dilemma is pregiven would be an anathema to most modern philosophers. The aim of a lot of modern philosophy is to examine as far as possible, the arguments for and against a given position. Often in modern philosophy one is forced to see both sides of the argument without any definite conclusion being made and a lot of philosophy involves either pointing out flaws in other persons arguments or modifying a previously held position in the light of other arguments and it is an ongoing debate. By this dialectic it is hoped that a more accurate position can be found or at least bad arguments for or against a given position will be rejected. Whilst the existence or not of God forms one part of philosophy it does not concern itself just with this issue or other religious matters.

It is true that with the rise of scholasticism in the middle ages that theologians such as Anselm and Aquinas saw the matter in a different light, they adopted the slogan 'Fides Quarens Intellectum' i.e Faith informed by knowledge. Their starting point was their faith and they then tried to use philosophical reasoning to inform and enhance their faith. It would be fair to say that most theologians still adopt this maxim. There is of course a degree of overlap. However whilst theologians look to philosophical arguments to bolster their faith, philosophers try to avoid all issues of faith when developing their arguments. A philosopher would be interested in such arguments as to whether or not there is a first cause behind all things (the cosmological argument). Whether or not the idea of a perfect being must include its existence by necessity otherwise the being would not be perfect (the ontological argument if I understand it correctly). Or finally whether an argument from design is viable. Modern philosophy of religion in the light of developments in the philosophy of language, also deals with the nature of religious language and whether or not it refers.

This branch of philosophy is called philosophy of religion, by philosophers and natural theology by theologians. Kant in his critique of pure reason put a large nail in the coffin of natural theology when he showed that the traditional arguments for God, from Design, the cosmological argument and the ontological argument led to contradictory conclusions. Thus for Kant at least, the use of reason left the question of the existence of God as undecidable. In my own opinion then philosophical arguments for the existence of God are at best inconclusive.

However even if natural theology could prove the existence of God, many theologians would see this as a starting point, but they also have their own agenda, it would be a pretty poor conception of God, from a theologians point of view that didn't talk about his/her/its intervention in the world. In terms of traditional Christianity anything which didn't mention the alleged fall, and wasn't centred on the teachings or significance of Jesus Christ for redemption would be a pale reflection of what they see as the core of their message. Indeed there have been theologians such as Karl Barth who reject natural theology altogether. Given the problems associated with natural theology as outlined above one might have some sympathy, but to then claim a special type of revelation through either scripture or revelation would be a step to far for most philosophers. Again theologians of other religions would want to supplement the arguments from natural theology with their own perspective. However by doing this they move away from philosophy to theology and as each theologian is usually tied to a specific religion their arguments lack universality.

To get an idea of the contrast compare Hans Kung's two books 'On being a Christian' and 'Does God Exist'. On being a Christian concerns itself with Christian doctrine and what it might mean for today. Whereas 'Does God exist' concerns itself with philosophical arguments for and against the existence of God as they have raged down the centuries. Hans Kung comes to the conclusion, along with Pascal, that the God of the philosophers is not necessarily the God of theologians. To some extent the approach of a philosopher to the existence or otherwise of God is complementary to that of a theologian but both have quite different agendas.

As for the second part of your question (and what follows is my own opinion ), in general religion is tied to a particular institution, which wishes to see its own agenda adopted by the populace at large. In theocracies, which thank goodness we currently do not live in, (although some parts of America come pretty close), the teachings of a particular religion, are imposed on the whole population irrespective of whether or not they believe in the religion. There are vested interests in both the Catholic and Protestant churches which would like to see such theocracies imposed in the West. In my opinion a lot of what passes for the public face of religion is just bigotry (such as policies against gay people, the control of women and campaigns against the abortion laws). Again whilst it may be tolerated for priests and bishops to study theology, in general the church does not encourage its lay folk to do so. I for one think it scandalous that two hundred years after the development of Biblical criticism, people in the church for the most part are encouraged to treat the Bible or Koran as if it were the literal word of God rather than a piece of literature. Thus whereas a philosopher would treat the moral precepts of the Bible or Koran as rules of thumb subject to revision and would want to look at the arguments (if any) for the particular moral precept, religious institutions on the whole sees them as eternal unchangeable truths and as such do not consider the arguments for and against the particular ethical position.

On the other hand spirituality is more concerned with an individuals experience, a sense of awe and wonder at the world around us which is exploited by the Church and other religious institutions to persuade people that this is an experience of God. In my mind it would be better to describe spiritual experiences as aesthetic and not leading to God as there is no hidden agenda. For me the arts and sciences provide a much more satisfying method of fulfilling my deepest needs than anything I can get from institutionalised religion.

Christopher Finlay

back

(52) Al asked:

I have a question with regards to what sort of structure to use when answering a particular philosophy question. The question posed to me is as follows.

'Thank you very much for your thoughtfulness' If Darwinism threatens our ordinary ideas of responsibility and altruism, how should we understand this phrase and others like it?

I understand that this is a conditional but am struggling with exactly how to present my answer.

---

I would break the question into two parts, given that, as you say, it's a conditional.

(1) Is the antecedent of the conditional true? (If it's not, then the truth of the conditional is trivial and so completely boring.) So you need to talk about whether Darwinism really does threaten our ordinary ideas of responsibility and altruism.

(2) Assuming that the antecedent is true (even if you have argued that it isn't), does the consequent follow? Well (reading 'threatens' to mean 'undermines') that's another big question. What it is to be 'thoughtful'? Does thoughtfulness imply altruism? Or can people be more or less thoughtful (e.g. not acting impulsively and considering the possible harmful consequences of their actions) even if whatever they end up doing can't be thought of as 'genuinely' (whatever that means here!) altruistic?

I hope this helps!

Helen Beebee

The British Philosophical Association
http://www.bpa.ac.uk

back

(53) Royce asked:

Me and a friend are discussing the definition for atheism. He is a theist and I am an atheist. He (in my opinion) is confused about the definition. But I would love it if you can please tell me how to define it for him. Thank you.

---

Well, I would say an atheist is someone who believes that God doesn't exist. So that's pretty straightforward! Note that atheism doesn't require that you're absolutely certain that God doesn't exist. I believe that the external world exists, so I'm a realist — but I accept that I might possibly be wrong about that. That doesn't stop me counting as a realist. Of course you can now start complicating matters by asking how strongly you have to believe that God doesn't exist in order to count as an atheist, if you think that there can be different strengths of belief, ranging from not believing something at all to being absolutely 100% certain. But that's really an issue about belief (how strongly do you have to belief anything in order to count as positively believing it?) and not about atheism in particular.

Helen Beebee

The British Philosophical Association
http://www.bpa.ac.uk

back

(54) Sahr asked:

The ancient Greek philosopher Plato was a Socrates' student and also an Aristotle's teacher.So he could be called either a 'student' or a 'teacher'. My question is who really is a 'student'? Can University Lecturers be students if they so desired?

N/B: I Would love to reference this answer in my conversation.

---

Plato was both a teacher and a student: they are not mutually exclusive. A good philosopher is a student all his life , since there is always something new to learn. I myself may well not be a good philosopher, but while I was a professor of philosophy I used to take undergraduate courses in math and physics at my own university. If you want to reference this, my university was the University of Guelph, in Ontario, Canada.

Helier Robinson


back

(55) Lauren asked:

If you force sex upon a prostitute, is it rape... or stealing?

---

It is always rape by definition i.e. the key word here is 'force'. In any country where verbal contracts between a prostitute and a client are recognised as legally binding then a refusal to pay after voluntary sexual intercourse with a prostitute would be breach of contract. It would never be theft.

Shaun Williamson

back

(56) Rocky asked:

Today as you know we face lot of problems regarding different religions in the world. My view is that people should take what is good in every religion and be friendly with one another. Build a system of friendly religion or a unifyng religion. I would like to know which existential or postmodern philosopher upholds my idea so that I can do some research on it. Kindly help me.

---

I'm sorry, Rocky, I'm not going to answer your question in the way that you would like. But that is not because I think that the very idea of 'friendly' religion where all would share the same faith is an absurdity — given what we in fact know about the world religions. Arguments from the facts, no matter how seemingly obvious, are always suspect in philosophy. From what is you cannot logically deduce what can or cannot be.

Perhaps on some planet somewhere — or perhaps on Earth 2000 years in the future — there is a universal religion such as you describe, where all human beings join together in joyful worship of the 'one true God', where there are no warring sects, no hostility, only friendship and love. (As it happens, that is not so very far away from what Catholics hope and believe. Perhaps in Islam too, the ideal state would be reached when there were no more 'infidels'.)

Let's just take that as read. From the tone of your question, I gather that you would find that picture attractive. I have no comment to make about that. What I want to do instead is look deeper into your question, picking up on the reference to existential thought which is the key here.

The first thought is whether there is something we can identify as the true 'core' of all religions which deserve the name. To have a religion is to worship something, which in turn implies that some entity is regarded as worthy of worship and indeed the only such entity. The name we give to this entity is 'God'.

Already this seems to jar. Zen Buddhists don't 'worship' a 'God'. Should Zen Buddhism therefore not be categorised as a 'religion' but only a 'philosophy'? or something else? But this looks like nit-picking, perhaps we can ignore semantics as a side issue and concentrate on the main question and see how it relates to existential thought.

There are two important ideas that one finds in existentialism. The first existentialist, the Danish philosopher Kierkegaard, claimed that the religious attitude is characterised by a leap of faith. There cannot be 'evidence' for faith in this fundamental sense. Historical claims like the resurrection, or proofs of God offered by rationalist philosophers are irrelevant. One can go further and state that what this God is, is in an important sense irrelevant also. The content of faith is in the actions which it inspires. If your faith inspires you to imitate the example of Christ, then you would call yourself a 'Christian'.

The second important idea comes from Nietzsche. Stated very simply and brutally, Nietzsche's idea is that God is dead. Although this seems like as close as you could get to atheism, this thought inspired a major strain of 20th century Protestant theology. Just as we cannot find empirical or rational evidence for God, so nothing has value for us unless we value it. It is ultimately our responsibility to choose the kinds of values — the kind of life — that we find meaningful.

Both ideas come together in the thought that what is central to religion is negative rather than positive, not 'belief in God' as an entity or in values written on tablets of stone, but rather the rejection of false gods. Perhaps one of the most potent arguments for faith is that those who lack it are doomed to the futile pursuit of false gods.

Well, if that is so, then it seems to me that there is only one more step that needs to be taken. The only true religion is one which rejects all gods, period. Anything you say about your 'god', any belief or value you associate with him, or it, is false. As Wittgenstein remarked (in relation to a different question) 'a nothing would serve as well as a something about which nothing can be said'.

The children of Israel were aptly so-called. Jews today celebrate the escape from slavery in Egypt and later their leader Moses destroying the golden calf, two of the most important steps taken by the human race towards adulthood. Yet children they remained. (The same, of course, applies to Muslims and Christians for this is their story too.) So long as there remains an object of worship, however abstractly we conceive it, human beings will remain essentially children, fantasizing a family story of a loving parent and his obedient, or disobedient, offspring.

I am tempted to say, 'It is time to put aside childish things and take responsibility for our own existence.' Yet I wonder if that is true. I wonder whether human beings are, or will ever be capable of taking that step. It is easy enough to declare oneself an 'atheist', or 'against religion', much harder to embrace the full consequences of that existential choice.

Geoffrey Klempner

back

(57) Kogure asked:

If you cannot be certain on what the truth is based on experience, can you know anything at all?

---

Yes, you can. You can be certain of the Cartesian cogito. You own existence is certain, since you have to exist in order to ask your question.

Helier Robinson


back

(58) Glenstein asked:

In Sec. 185 of Wittgenstein's investigations, he gives an example of a student who is being to taught to count by increments of two. He does fine up to 1000, but after this point, he starts incrementing by four, and we say to him 'look what you've done!'

The student thinks he did nothing wrong, and imagines that his task was to increment by two only up to 1000, and then increment by 4 up to 2000, by 6 up to 3000, etc. etc. I believe Wittgenstein's point was that, when you teach a 'rule' by giving examples of its proper use, you can fit any number of behaviors to that rule.

Well, I just completed a 'Mensa Fun Test', where they frequently ask 'what comes next in this series of numbers?'

One example gives the following series:

2, 3, 5, 7, 11, 13,??

I got this correct, answering 17. 17 was the 'right' answer because this was a series of prime numbers.

But for a moment, I mistakenly thought 15 would come next because I thought I was looking at some sort of adding pattern, not unlike Wittgenstein's student. First you add one, then you add two, then you add two again, then you add four, and then add two again.

In retrospect, I don't think there is any justification for answering of 15, even if I had established some coherent adding pattern. The fact that these are primes is certainly more obvious. But, would I actually be wrong to answer 15? It vaguely seems like there is something ridiculous about insisting that 15 could be right, because it accords with some series that was a legitimate alternative to the prime number series.

---

Well first I have to say that you have completely misunderstood the point of Wittgenstein's remarks and you have misunderstood the nature of intelligence tests.

Wittgenstein emphasized that his remarks only have meaning in the context of the particular philosophical problems that he was discussing, in this case what philosophers say happens when we can be said to be following a rule.
You are not meant to take any of his remarks in isolation or to imagine that they mean anything outside of philosophy. You cannot apply them to psychology and Wittgenstein didn't intend them to be used in this way.

Now to intelligence tests, people often mistakenly think that intelligence tests are based on questions which have a right or wrong answer and they feel entitled to argue that surely this other answer is equally valid.

Intelligence tests are never based on the idea that one answer is the right answer to a question. They are based on the idea that intelligent people give this answer to this question while less intelligent people give other answers.

You need to study in detail the history of how intelligence tests evolved and how they are constructed. There are many reasons to criticise intelligence tests but yours isn't a good reason.

Shaun Williamson

back

(59) Richard asked:

In New Zealand we have groups of young people who hoon around in cars making lots of noise and pulling doughnuts. They are called 'Boy Racers'. They modify cars illegally and are disrespectful to authority. (Arguably they are like the Futurists in that they celebrate speed and technology). My question is can a typical boy racer be justified in seeing himself as living a virtuous life . Is he being unethical?

---

One of my favourite NZ words is 'hoon' although its meaning might not be obvious in the U.K. I think it corresponds to our words such as yob or lout. Also 'pulling doughnuts' is a wonderful phrase. I will have to get my NZ wife to translate that one.

We also have boy racers in the U.K. and I am sure that they exist in other countries. The futurists may have celebrated speed but they didn't insist that they were entitle to race up and down main street, putting other people in fear of their lives. I am sure that many boy racers are ethical and potentially good people who are virtuous. However like many adolescents who are becoming adults but still acting like children, they can be a hazard to the rest of society.

So yes, a boy racer can see himself as being a good person but if he won't stop illegally racing up and down main street then confiscate his car and hope that he will some day understand that other people can be leading virtuous lives also and that they don't want to be run over by a reckless boy racer.

Shaun Williamson

back

(60) Don asked:

I have been trying to educate myself about Mtheory (i.e. the potential theory of everything) and I was wondering if there has been any work that tries to make the connection between consciousness and a theory of everything. What are the philosophical cosmology implications of interjecting consciousness( or mind) into the discussion. I recently read J.r. Searle's 'Mind' and began to wonder if matter responds to consciousness and if matter and energy are related, does consciousness cause energy and matter. If so ,what are the implications for Mtheory? Any feedback would be appreciated. I am a psychologist by training but interested in the big questions. Thanks,

---

When physicists talk about a theory of everything they usually mean a theory of everything in physics, and, perhaps, chemistry. If you had such a theory and extended it to include mind, you would have a philosophical system rather than a physical system: a much more desirable result I think (speaking as a philosopher). So I certainly agree with you about needing to interject consciousness into the discussion.

Certainly some matter responds to consciousness: your material muscles respond to your willing them to move; but I do not think that this means that consciousness causes energy and matter. It is fairly clear that the human body is an energy system, since it needs food (calories) to continue to function; but whether the mind is also an energy system is less clear. Since you have a PhD I am sure that you know from experience that long, concentrated, mental work is very tiring, which suggests that energy is used in the process.

But although energy is necessary for consciousness, it is not sufficient, since otherwise steam engines and the electrical grid would be conscious. My own view is that mind is an emergent out of brain and as such possesses properties not possessed by brains or any other lower level systems. Other examples of emergents are the working order of a machine when it is correctly assembled, or the melody emergent out of a collection of notes when they are properly arranged. Equally, life is emergent out of chemistry. In this approach it is a reductionist error to suppose that life IS chemistry, or that consciousness IS brain, or that working order IS the parts of the machine. If you are interested in more of this, I have developed a modest philosophical system using this approach, which can be found at www.sharebooks.ca. (This site is a bit of a mess at present, but it is being upgraded and the new one should be available soon.)

Helier Robinson


back

(61) Jason asked:

How much intelligence does a philosopher need? i'm majoring in philosophy, I enjoy it, but I wonder whether I can comprehend some of the larger concepts. it's silly to put it this way, but can it be summed up into an IQ number?

---

No, the study of philosophy cannot be summed up as an IQ number. Obviously some intelligence is required in philosophy, but curiosity and motivation are much more important — unless, of course, you want to make a major contribution to the history of ideas, in which case genius is required.

Helier Robinson


back

(62) Don asked:

I have been trying to educate myself about Mtheory (i.e. the potential theory of everything) and I was wondering if there has been any work that tries to make the connection between consciousness and a theory of everything. What are the philosophical cosmology implications of interjecting consciousness( or mind) into the discussion. I recently read J.r. Searle's 'Mind' and began to wonder if matter responds to consciousness and if matter and energy are related, does consciousness cause energy and matter. If so, what are the implications for Mtheory? Any feedback would be appreciated. I am a psychologist by training but interested in the big questions. Thanks,

---

I think there is a danger here that you are being deceived by media hype. There is no theory of everything and no one is looking for such a theory.

In physics the phrase 'theory of everything' is used to refer to an, as yet unknown, theory that would provide a unified explanation of the four forces that affect matter. At present physicists have two main theories to explain things, quantum mechanics and Einstein's theory of Relativity. Quantum mechanics deals very well with the atomic and sub atomic aspects of matter, Relativity deals with the large scale things such as space, time, planets, stars, galaxies, quasars and black holes.

The four forces are:

1. the electro-magnetic force
2. the strong nuclear force
3. the weak nuclear force
4. gravity

Quantum mechanics deals very well with 1,2 and 3 but so far does not provide a framework for understanding gravity. So at present physicists are searching for a sub-atomic particle known as the Higg's boson (which has unfortunately been nicknamed the god particle). If they find the Higg's boson then this will help to incorporate gravity into quantum mechanics and will make a 'theory of everything' possible.

However in the background another theory is lurking that may replace both Relativity and quantum mechanics. It is called string theory. At present physicists are making precise measurements of the orbit of the moon to see if the results are more in accordance with the predictions of Relativity of string theory.

However none of this has anything to do with consciousness, physicists aren't interested in consciousness. People who try to spin fanciful theories connecting consciousness and quantum mechanics are not scientists. They are talking nonsense.

So you wonder if matter responds to consciousness. The answer to this is no it doesn't and it can't. My body is made of matter, that is atoms and sub atomic particles. I am conscious. I control my body. My consciousness doesn't control my body. I have a mind. My mind doesn't control my body, I control my body.

People have material bodies and people control bodies, unless some pathological condition makes this impossible. Minds and consciousness are not the sorts of things that have bodies or control bodies or interact with bodies. Once we start dealing with complex abstract concepts it is easy to weave all sorts of improbable fictions around them and difficult to maintain a clear idea of the differences between all these things.

Shaun Williamson

back

(63) Jason asked:

How much intelligence does a philosopher need? i'm majoring in philosophy, I enjoy it, but I wonder whether I can comprehend some of the larger concepts. it's silly to put it this way, but can it be summed up into an IQ number?

---

No you can't sum it up in an IQ number. When you are studying any subject it helps to have a high IQ but other things like interest and talent are also important. No matter how high your IQ is, if you don't have a talent for mathematics you will never get a good major in mathematics. This is also true for English and Philosophy.

Of course philosophical concepts are difficult so are the advanced concepts of mathematics, economics, psychology. All subjects that you study in higher education are difficult.

Why did you decided to major in philosophy? Do you enjoy the subject? These are the sorts of questions you should be asking yourself. There are no easy and no hard subjects at university or college level. all of them are equally hard but avoid subjects that don't interest you and that you have no talent for.

Shaun Williamson

back

(64) Andrew asked:

Is wonder the best thing to do?

The International Society of Philosophy makes this claim Wisdom begins with wonder I would go further because not everyone seeks wisdom.

Wonderful.

OK Im dying and you say I can have one last thing before I die. I would not say can I have more wisdom. I would say can I have a bit more wonder please and hopefully my last word will be WOW!

The universe shows itself off to me and I am glad.

So a reasonable way to live would be to cultivate a sense of wonder. In order to recognise a wonder when you see one.

It would also be a good idea to have a few wonders stashed away just in case you happen to be dying one day and are in need of one.

---

There are different sorts of wonder. If you have never tasted good food then you might find McDonald's wonderful. You might even say 'wow' when you taste your first Big Mac.

True wonder can only come with wisdom because then you will know what is truly wonderful. If you just want to say 'wow' then take lots of psychedelic drugs just before you die.

The universe only show its true nature to those who have wisdom. If all you want is wonder then you are easily pleased and stay away from philosophy. Philosophy doesn't need people who are easily pleased.

Shaun Williamson

back

(65) Louie asked:

Is it possible that there could be a overwhelming stream of information, via the five senses, that bombards us throughout our daily routine; this stream cannot all be deciphered/ processed, so eventually the subconscious brain acts as a filter. Filtering out the "useless" and processing the "useful". And that all humans are innately desensitized to their environment?

---

Bertrand Russell's collaborator A.N. Whitehead, at the beginning of his magnum opus Process and Reality comments that human beings habitually 'perceive by the method of difference'. We notice an elephant in the room because an elephant is not always there. Whitehead is speaking figuratively. If an elephant followed you everywhere you'd still be able to 'notice' it whenever you wanted to! But Whitehead is talking about general and structural features of our experience, the kind that interest the philosopher engaged in metaphysical inquiry.

The inhabitants of Alex Proyas' Dark City (1998) never think to ask themselves, 'When was the last time you remember doing something in the day time?' Could we be like them? Might there be something in our world, and not just in its general or structural features that we fail to see because our senses are overloaded and we've become desensitized?

This is one of the perennial themes of radical left politics, the idea that we are living in a state of perpetual 'false consciousness', our minds numbed into paralysis by insidious propaganda. If you're interested in science fiction, then Dark City or The Matrix provide models for all sorts of fanciful conspiracy theories.

Anything might be the case, in some possible world, if you are prepared to stretch your imagination far enough. But that's not what the question is about. The question is about how things might be for all we know, that is to say, how they might be in the actual world, right now, and we would never know the difference.

This question grips me. I think I know my world. I'm pretty confident that I have a fairly useful grasp of science, politics, history, geography — the kind of stuff you'd call 'general knowledge'. But what if I'm wrong? What if there's something really huge that I'm missing. As I said, this isn't about science fiction or conspiracy theories, it's about reality, 'how things are right now'.

I don't have an answer. It's an 'unknown unknown'. I don't have the parameters for making a judgement about what is here, all around, that my senses and my intellect don't notice because it's all around, swamping my capacity to sift for information.

Good question, Louie.

Geoffrey Klempner

back

(66) Don asked:

I have been trying to educate myself about Mtheory (i.e. the potential theory of everything) and I was wondering if there has been any work that tries to make the connection between consciousness and a theory of everything. What are the philosophical cosmology implications of interjecting consciousness( or mind) into the discussion. I recently read J.r. Searle's 'Mind' and began to wonder if matter responds to consciousness and if matter and energy are related, does consciousness cause energy and matter. If so ,what are the implications for Mtheory? Any feedback would be appreciated. I am a psychologist by training but interested in the big questions.

---

I can only give my opinion as a practising physicist who has no real knowledge of M-theory, but I really feel that there is no connection between consciousness and M-theory. It is difficult to see M-theory as a 'real theory' of physics for the following reasons:

1) There is no definitive formulation of the theory. 2) It seems to be more of a conjecture than anything else. 3) Most crucially it is not clear that there exists any real connection between the theory and what can be measured in the lab. Thus until such time as it can produce any quantitative predictions it is to use Dirac's damning phrase 'Not even wrong.'

These points are developed at great length in the book 'Not Even Wrong' by Peter Woit amongst other books. One thing that a lot of people find difficult to accept is the fact, that despite the hype, so called 'theories of everything' do not really have much application in the 'real world'. To solve most day to day problems in engineering or physics, classical physics still forms the basis of much of current design work. Designers of radar such as myself use classical electromagnetism as the basis of how radar systems work, quantum effects, if any are so small as not to affect the sorts of phenomenon we are interested in. Thus it is not clear that looking for a solution to a phenomenon such as consciousness which is a macroscopic phenomenon, from say the latest developments in string theory has any merit or relevance.

However the question still arises what is the relationship if any between consciousness and physics. This is a matter of controversy as I'm sure you are well aware. In fairness I shall outline some of the main areas where the link has been made and then give my reasons for rejecting or accepting some of them.

1) The theory of quantum measurement as formulated by Von-Neumann and developed by London and Bauer. Quantum states describe the possibilities open to a system, the mathematical function describing this is called a wave function. However when a system can be in a number of states (eg an electron can have two possible spin states) the wave function describing this is a superposition of both of these states. However the electron is only ever observed in one of these states when a measurement is performed. Thus the wave function has collapsed into one of the possible states describing the system. The resolution of this problem which seems to be an extra ingredient to add to quantum mechanics is called the quantum theory of measurement and the question is what is the significance of the so called collapse of the wave function. One view, which seems to be the one favoured by popular books, is that it is human consciousness that is ultimately responsible for collapsing the wave function. This 'California' interpretation as I call it really stretches credulity. There are a number of responses.

a) Denial that the wave function collapse is a real event those who favour the statistical approach to quantum mechanics (such as myself) do not believe that the wave function is anything more than a heuristic device for calculating probabilities, thus the wave function should really be called a probability state vector and not a wave function.

b) However even if the wave function collapse is a real process, it is not clear that consciousness plays a part in quantum measurement. Niels Bohr was adamant that long before the results of a measurement took place an irreversible amplification of microscopic events occurred (eg for a photon hitting a photographic plate) before a human observer observed the results. He/She certainly does not create events by an act of observation as the California interpretation would claim.

c) Finally however there exist other interpretations of quantum mechanics such as David Bohm's or Everrett's Many worlds interpretation which attempt to remove the notion of wave function collapse from quantum mechanics.

Thus it would seem, despite some claims in the literature, that wave function collapse, if it is a real process has little to do with consciousness.

2) The application of quantum gravity to consciousness. Roger Penrose in his books 'The Emperors New Mind and 'Shadows of the mind has claimed that quantum gravity is needed to 'collapse the wave function'. However in the absence of any definitive formulation of quantum gravity, it is not clear that this is any more than speculation.

3) The use of chaos theory, to my mind, this seems the most promising (if not the most sexy) application of physics to the problem of consciousness. Chaotic systems are essentially unstable classical systems in that it just takes a small change in the variables to produce completely different behaviour. However within the unstable environment there are islands of stability. Work by such people as W. J. Freeman (see link below) has spent a long time investigating these phenomenon and there does seem to be strong evidence that chaotic phenomenon enable the brain to jump from one state to the other and is necessary for say perception in the form of smell. Whilst this is not a fully fledged theory of consciousness, the fact that the results of such an approach are able to be tested empirically makes this approach much more promising and well defined than the other approaches outlined above.

As a final point, whilst all this speculation is interesting, even if we could have a convincing explanation of consciousness in terms of the latest physics, it is not clear that would that solve the problem of consciousness in a way which would satisfy our own intuitions. A vexed problem is the problem of qualia i.e its all very well having say a good general understanding of the physics of how we perceive colour but this general description would not necessarily capture the subjective aspect of colour as I myself perceive it. So in conclusion it may well be that physics is able to help us understand some aspects of consciousness better but I remain sceptical that it will capture all the aspects of consciousness we require for a full understanding.

Further reading

A general survey of the problem of quantum measurement, from someone who is currently championing the Everrett interpretation is provided here.

http://users.ox.ac.uk/~lina0174/philphys.doc

Freeman's work on the relevance of chaos theory to understanding the brain is accessible via this web site

http://sulcus.berkeley.edu

I suggest starting with the paper Consciousness Intentionality and Causality accessible by clicking on the WJF Neuroscience Articles button on the web site above.

I have given some quite sceptical comments about the relationship between quantum mechanics and consciousness, for a more positive view see eg

Danah Zohar The Quantum Self: Human Nature and Consciousness Defined by the New Physics

Christopher Finlay

back

(67) Lina asked:

I've just begun reading Plato's Gorgias and was wondering to what extent is Socrates's dialogue a representation of Plato's views?

---

This is a difficult question to answer and perhaps ultimately insoluble. The general consensus seems to be that Socrates in the Gorgias is at least somewhat representing the author's views. The Gorgias (along with others like Meno and Phaedo) is often thought to be one of Plato's bridge dialogues -a transitional dialogue from (the supposed historical) Socrates' views of the early Platonic works (like Apology, Laches, Crito and Protagoras) to Plato's own mature views (as mouthed by Socrates) in the later works (e.g. Republic). The reason many scholars think this is the case is because Socrates is made to say some important things in the Gorgias that seem to contradict what he says in Plato's earlier dialogues.

A couple of examples will have to suffice. In the Apology Socrates claims ignorance as to the existence and nature of the afterlife ; however, in the Gorgias he speaks (to Callicles) with assurance and authority about a myth of judgement after death (similar to that found at the end of the Republic) -in fact, he says he believes it. Another example concerns Socrates' moral psychology. In the early dialogues, Socrates believes that we are basically all rational; in the Protagoras he says 'knowledge is something noble and able to govern man, and that whoever learns what is good and what is bad will never be swayed by anything to act otherwise than as knowledge bids' (352c). However, in the Gorgias Socrates seems to believe in a set of unruly desires located in a separate part of our soul which can trump our knowledge of what is good and lead us away from it. This clearly anticipates the model of the tripartite soul presented in the Republic, and suggests to us that Plato is no longer representing Socrates' views.

Kristian Urstad

back

(68) Robert asked:

I am a self taught hedonistic cynical minimalist philosopher with an engineering degree. It seems the vast majority of humans are content with 95 for 40 years to pay off a house on a postage stamp sized lot. I am not. I worked for ten years, and then engineered myself a second childhood raising my two sons full time. I built a sanctuary like bubble and lived a non materialistic and hedonistic life , playing games with my sons while the world went about its business. An air strike was called in last October, decimating my bubble and separating me from my kids. I was thrown into society, with nothing similar to Pangloss in Voltaire's work Candide. What do you recommend I do. It has been 8 months since I was B52 bombed, and I still feel lost.

---

I wonder how we are supposed to interpret your story? I don't understand it, so I wish you had been more explicit. However what seems to be true is that you were busy spending your time raising your two sons and now you are not and have nothing to do.

Of course two young children will keep an adult very busy and not leave much time to think about anything else. Now you have nothing to do and plenty of time to think but nothing to think about. So it is not surprising that you feel lost.
No one can tell you what to do but you need to find some goal that means something to you and that you can work towards. Get a job, write poetry, write a book, retrain as a teacher and work with children. Do anything but do something, if you don't you will not survive. If you are unable to do anything then you should get some help, some sort of counselling. It is possible that you are depressed and suffering from grief over the separation from your children. If you need help then don't be too cynical to seek it out.

Shaun Williamson

back

(69) Carmen asked:

how can I get people to do what I want them to do?

---

Buy a big gun. Failing that try finding someone to love who loves you. Then you won't want to make make other people do what you want them to do because you will be a full grown up adult person.

Shaun Williamson

back

(70) Alex asked:

I am the organizer of the NYC Existentialist Reading Circle. I am new to this philosophy and need to learn much.

I have a member who is trying to convince me we need to read Marx. Can relay the significance of Marx to existentialism. I understand Sartre was involved with this.

---

Marx was not an existentialist, you do not need to study him in order to study existentialism. Marx isn't even all that interesting as a philosopher. He is an important political thinker, sociologist and a moralist. Also many of Marx's major works such as Das Kapital are long winded and boring, so don't even try to read them.

Sartre was an existentialist who later became a Marxist. Heidegger was an existentialist who later became a Nazi but that doesn't mean that Hitler has any relevance to existentialism either.

Shaun Williamson

back

(71) Matas asked:

I am a 16 years old boy from argentina, and I would like to ask, in what a philosopher may work. And where I can study for being a philosopher.

---

A philosophy degree is an arts degree so it is as useful for employment generally as a degree in Spanish or history or sociology. The only job specific to philosophy is being a philosophy lecturer at a university. These jobs are scarce and not easy to get.

You study to be a philosopher by going to university and doing a degree in philosophy. Before you decide to do this you should read some philosophy books to decide if it is a subject you really want to study.

Shaun Williamson

back

(72) Erica asked:

I have 2 questions...

1. What exactly is a soulmate, and do they actually exist? I have searched in several dictionaries and books, but I would be interested to know more details.

2. If there is such a thing as soulmates, are they still connected to us after death (if there is an afterlife )?

---

To go direct to the source on this one you need to read a very short dialogue from Plato called the Symposium. That's greek for drinking party, so don't expect any academic jargon or management speak.

Plato's so much a part of modern consciousness that people don't know when they are spouting him. And it will help to see the thought in it's origin context, if you want to go on to criticising it. I suppose having read the Symposium, if you want more clarification, you'll find it in other dialogues like the phaedo, phaedrus, republic etc.

David Robjant

back

(73) Jake asked:

What is the purpose of aesthetics? Why are we so moved by poetry, music and art? And why do our tastes of such things differ from one person to the next? None of these are necessarily essential for survival, yet we hold them in high regard in our life . Furthermore, our tastes for such things change over time. Can this be explained in a logical sense? Is man a rational animal to hold these things in such high esteem? How are these things pertinent to us in our day-to-day individual fights for survival we call life ?

---

I can't resist referring you to your namesake in a novel by the name of 'Under the Net', by one Iris Murdoch. That Jake has an argument with a certain Hugo. Hugo, rather like you perhaps, finds our addiction to myths and images and theories and all of what you might call 'art' something irrational and despicable, something that turns us, indeed, into liars. Jake responds that myth and theory and image are how we understand the world, and that 'art' therefore is reason.

In fact, with your worries about the moral and epistemic status of art, I can't think of any publication by Iris Murdoch that you wouldn't find touching on your concern. But I would recommend you get hold of the collection 'existentialists and mystics' first, and head first for the script of her play 'Art and Eros'. That is, after reading 'Under the net'.

By the by, I think it is quite a common theory with those interested in such matters that the development of Art by the human species (cave paintings, carvings, etc) is fairly closely connected to our remarkable survival. After all, fangs winter fur and deadly speed we do not have. What do we have? Art and language. The argument then will be about whether there might be some kind of language where no art is, and how useful that would be. George Orwell reminded us that talking good sense, and understanding what you yourself are saying, is closely connected with mastery of the lively 'concrete' image — attempts to do without it come out with garbage. A certain way of looking at science suggests to us here an area of talk without art and imagery: but the suggestion is wholly misleading. Think of Einstein and his trains. Imagery is a crucial part of thinking, that is to say of reason, at every level.

Tastes change, you say, and I think you are implicitly arguing that there is no disputing about taste, and that therefore there is no knowing or truth where there is taste, and art must be irrational. Well, as someone once said, so far from there being no disputing about taste, I suggest that taste is the only thing worth disputing about. A good scientific theory will be a beautiful one. Ugliness goes with accumulations of ad hoc explanations and work-arounds, beauty with a nice simple clear picture of the thing that can be put to work with ease.

So much for art. As far as the point of aesthetics is concerned, well, there you have me.

David Robjant

back

(74) Alex asked:

I am the organizer of the NYC Existentialist Reading Circle.

I am new to this philosophy and need to learn much.

I have a member who is trying to convince me we need to read Marx. Can relay the significance of Marx to existentialism.

I understand Sartre was involved with this.

Significance of Marx to existentialism: Death. Sartre's involvement: philosophical suicide.

---

It is perfectly true that Sartre tried to reconcile marxism and existentialism. I've not met anyone who believed that he managed it, but it looks like you have. I suggest you treat this person with every kindness other than letting him dictate group reading.

You say that you are new to this philosophy lark, but as organizer of the existentialist reading circle I would hope you've read Nausea (La Nausee)? If you want to read something very short and to the point about Sartre's political diversions, what Sartre thought he was doing and why it doesn't work, I suggest you read Iris Murdoch's 'Sartre: Romantic Rationalist'. There's also all that Camus stuff where he talks about the idiocy of the french Left. Is Camus an existentialist? More than Marx was, that's for sure.

David Robjant

back

(75) Frank asked:

Discuss the difference between versions of utilitarianism that regard 'pleasure' as the unit of utility, and those that take it to be 'desire-satisfaction'.

---

Utilitarianism combines two elements. The first is consequentialism, the view that one should act so as to produce the best consequences . The second element is the view — sometimes known as 'welfarism' — that goodness and badness (of the consequences of actions) is to be defined in terms of the well-being of human (or, in some versions, sentient) beings.

The difference between the versions of utilitarianism which focus on pleasure and those which focus on desire-satisfaction rests on a different conception of human well-being. At its heart is the fact that hedonists (those who focus on pleasure, or happiness) believe that ultimately well-being depends upon the quality of one's experiences or mental states. Desire-satisfaction theories, on the other hand, hold that states of the world — i.e. those which satisfy our desires — have value.

The position is in practice more complicated than this. There are different variants of both hedonism and desire-satisfaction theories, and some theories attempt to straddle the divide in one way or another. For example, many desire-satisfaction theories allow that pleasure and pain also have value in their own right, and one theory (see Sumner book below) defines well-being in terms of happiness, with the requirement that it be 'authentic', which implies a relationship between the subject's mental states and certain relevant states of the world.

The main challenge to the hedonistic approach is based on the commonsense intuition that the way the world goes does make a difference to our well-being, even if we are unaware of it. This is brought out in thought-experiments such as Robert Nozick's 'Experience Machine' — a machine which will provide any subjective experience we could possibly wish for. Desire-satisfaction theories, on the other hand, have to cope with cases where the satisfaction of desires appears not to enhance our well-being, and also with those where something not desired beforehand does appear to do so.

Despite their differences, both hedonistic and desire-satisfaction theories of well-being are regarded as 'subjective', since both make well-being ultimately dependent (albeit in different ways) on some or other of the individual's subjective mental states: pleasures and pains, desires and aversions. Other theorists — some of whom are also utilitarians — reject subjective theories of well-being altogether and seek to define it in more 'objective' ways. Since the notion of well-being plays a part in many different kinds of moral theory, the debate between hedonistic, desire-satisfaction and objective accounts of well-being has an importance that goes well beyond utilitarianism.

Some useful reading beyond the classic utilitarian texts:

James Griffin: Well-Being, Clarendon Press 1986

Wayne Sumner: Welfare, Happiness & Ethics, Oxford University Press 1996

Fred Feldman: Pleasure and the Good life , Oxford University Press 2004

Tim Taylor

back

(76) Oliver asked:

How does a brain construct an opinion?

---

When we do an opinion poll we ask people what they think, we don't ask their brains.

Brains don't have opinions, only people have opinions. From time to time other people may ask you 'What do you think about this?'. They will never ask you 'What does your brain think about this?' because brains don't think, only people think.

If you want to know how the brain works, study psychology and neurology.

Shaun Williamson

back

(77) Lara asked:

As a philosophy student myself I'm pretty sure I'm right on this one, but just to check...

My friend asked, 'do you get answers wrong or questions wrong?', arguing that if one provides an incorrect answer one has 'got the question wrong'. I responded by arguing that this is a misuse of the english language, as he is trying to put 'question' and 'answer' in a tautologous relationship. In failing to provide the correct response to a question, one gets the answer wrong. this does not affect the question itself the validity of the question does not depend upon the answer given. The only way you can get a question wrong is if you are the one asking the question.

Surely I'm right? Another viewpoint would be much appreciated!

---

Yes I agree with you. Your friend is just playing around with words. Sometimes people understand the question and get the question right but give the wrong answer.

Sometimes people give the wrong answer because they misunderstand the question. These two things are very different and shouldn't be confused or conflated.

Shaun Williamson

back

(78) Andrew asked:

I am a good gambler. I can guess the correct outcome more than 50 of the time when others think its a 50/50 chance.

One of my strengths is that I know that I do not know anything. I can easily be wrong. I am wrong a lot and I lose money when I am wrong. I hate losing money and I find it very painful but I carry on because the lessons I learn are more valuable than money. Qualitatively more valuable.

But I am right more often than when I am wrong. Even when I am wrong I always learn something. Because I know I can easily lose and I am not rich because I know how easy it is to lose a bet consequently I only play for small stakes. The proof that I am right more than I am wrong is the fact that over 10 years I have won more than I have lost. Only fractionally more but as time progressing the fraction has been getting bigger. I know this pattern can easily not continue but I know that it is a good bet.

I only bet on human behaviour. The more I gamble the more I learn about human behaviour. The more I gamble the more I learn about my own behaviour. I have lost a lot of money through behaving like a Pavlovian dog. The lessons to be learnt from this type of thing are not easy to learn and I often fail to learn them the first second or third time. I know what it feels like to have my behaviour controlled by deterministic forces I can now stop it. It is not easy to do and the temptation, instinctive drive and demonic possession is always there waiting for me. But I know it for what it is and I can stop it. I choose always to stop it. Sometimes I fail. Sometimes I behave like a programmed machine. But every time It happens I learn more about myself.

Is knowing what is a good bet a form of knowledge? One does not have to win for it to be a good bet. Some hunches and guesses are better than others aren't they? I can live without certainty its useless anyway. The knowledge of a good gambler is actually more useful than any other knowledge. One reason for this is that because of my reputation I can influence other people and actually increase the likelihood of an event happening simply by my saying so.

I can also influence the odds. This enables me gamble that an event will happen and that it will not happen. The trick is to bet that statement x is true at high odds and bet that it is false at low odds. This is an effective way of ensuring that you guess the correct outcome more often than not. It is why I am right more than 50 of the time on 50/50 bets.

Keep your knowledge my guesses are far more valuable.

---

Knowing what is a good bet is a form of knowledge but it is not the only form of knowledge. Knowing how to build a power station and make an electricity supply is also a form of knowledge. If we can keep our knowledge does that mean that you will give up using our electricity and our television sets? Do you know how to make a television set or a computer or do you just depend on other people with knowledge to do this for you. When you are ill do you depend on medical knowledge or can you cure yourself with a few hands of blackjack?

The knowledge of a good gambler is no good without electricity. Try playing poker in the dark. The knowledge of a gambler can't help people who need a heart transplant, only the knowledge of a skilled surgeon can do that.

Perhaps the designers of aeroplanes could use your guesses instead of making difficult calculations and testing things. Would you like to fly on a plane made by gamblers who are good at guessing.

Can you do mathematics by guesswork?

Your bombastic and self-satisfied remarks just show that you have no idea what knowledge is.

Shaun Williamson

back

(79) Ramo asked:

As per evolution, will we all get hairless in future? How will a 'now' human look in year 400.000?

---

An interesting question but best answered by an evolutionary scientist but even then it would only be guesswork. We don't know how the planet will will change over this time.

The last great influence on evolution was the movement of people into cities especially in the last two centuries. The lack of proper sanitation led to a high infant mortality rate and meant that only children with very active immune systems survived. This may be responsible for the great increase in auto immune diseases just as the lack of food during the 1930's depression may now be responsible for high rates of heart disease in middle aged people since their mothers may have lacked the right sort of food during pregnancy.

I have noticed changes in people since my childhood that could be the result of evolution but they could equally be the result of environmental factors. For example when I was at school I only knew a few children who suffered from asthma, now we are told that it may effect one in every three children.

I have also noticed receding hairlines in a much greater number of young men, is this genetic? I don't know. It is almost impossible to predict the course of evolution. There are still so many things we don't understand.

Shaun Williamson

back

(80) Gina asked:

What does this question really mean: which are you drinking? the water or the wave?

---

In a good poem, no doubt one or two of various uncounted possible connotations would be given to such a contrast, which would suggest itself naturally in the context, and the upshot might be something thought provoking and observant.

But out of such a context and merely as it stands, the question is deep sounding drivel.

There is a certain way with words that lets us play at profundity. One characteristic of bad poetry (and philosophy), is a magpie attempt to string together pleasing sonorous declarations. There are those who are trying to sound like their idea of how a poet speaks, and similar phenomena bedevil philosophy and morals.

True, the connection between habit and virtue being what it is, imitating virtue may sometimes lead on towards virtue. But there is more to being insightful, expressive or wise than merely sounding it. And trying to sound it may be worse. A concern with controlling one's own appearance (not least as insightful, expressive or wise) pulls in precisely the opposite direction to (and can displace) genuine virtue and truthfulness. To be truthful is to be concerned with a world quite outside oneself. In this light, such disgusting absurdities as ostentatious humility may be compared with the disease of poeticism as sharing a kind of self-regard.

Like Philosophy, poetry indeed requires the lively image to get at the truth. But in philosophy and poetry both, poeticism offers encouragement and licence to pseuds and sound-alikes who take the richness of words and imagery (and even a certain tone of voice) in poetic or philosophical truthfulness as the thing itself — like a cat looking at the finger rather than the ball. This is one good reason for suspecting poeticist conventions, not least the new ones. As if the mere avoidance of rhyme (or use of symbolic logic) suggested expression and insight. As if, indeed.

David Robjant

back

(81) Demetreus asked:

Descartes and Plato equate knowledge with complete certainty.

Do you agree that knowledge requires this very high standard?

If knowledge means being certain, is there anything we can truly know?

---

I note the resonance with various remarks in James etc, but the opening statement is, I suspect, completely false.

It might be said that Descartes equates knowledge with what cannot be rationally doubted, but that is not the same thing as equating knowledge with certainty, or even with taking certainty as a necessary condition of knowledge. We may be certain (psychological state) of things which in the light of reason should be doubted and uncertain (as perhaps Descartes himself is at the beginning of his inquiry) about things which cannot be rationally doubted.

It is true that Descartes does try to specify necessary and sufficient criteria for knowledge, but he does not list certainty among his stated criteria, and the detail and significance of Descartes stated criteria of clarity and distinctness may be somewhat obscure and is much debated (I am tempted to take these 'criteria' as a celebration of necessary optimism). James took it that Descartes criteria are clearly incompatible with his own preferred criterion, but this is by no means obvious.

The ascription to Plato looks similarly shaky. It is connected perhaps with the summary of plato in which for him only the Forms are perfect objects of knowledge, and about these objects only does he allow certainty. That summary suppresses the way in which Plato uses 'episteme' and 'techne' to make a distinction within what we call 'knowledge': that is, what he actually says is that only the forms (like triangles, numbers etc) are perfect objects of episteme. In making that contrast between the nature and certainty of our knowledge of mathematics on the one hand and the nature of our knowledge of (say) boat building on the other, Plato is saying something perfectly proper, and, indeed, rather suggestive of James's pragmatism (our knowledge of boat building is a practical affair embodied in successful skill or 'techne').

In his use of 'episteme' and 'techne' Plato is not equating what we call 'knowledge' with complete certainty. He is quite rightly supporting the claims of mathematical knowledge (and other kindred 'episteme') to a distinctive status, an immunity to certain kinds of failure suffered in 'techne'. We aren't going to discover, for example, that sums calculated on a certain kind of timber do not last in sea-water. True, there are senses in which for Plato the knowledge of the forms etc is a 'higher' than technical knowledge, but the reasons for this are complex and do not amount to his thinking, in our terms, that episteme is knowledge proper and techne is not. One element of his thinking is that moral knowledge is 'higher' in the sense that thinking about what goals to pursue must come be prior to and in this sense 'higher' than (technical) thought about how to realise these goals. Plainly, Plato cannot think that knowledge of ends is valuable, and at the same time think that the knowledge or skill by which we gain such ends is worthless. There is no evidence that he did.

David Robjant

back

(82) Judy asked:

What role does justice play in postcolonial philosophy? Why was an education important for postcolonial thinkers? How did financial consideration and the international market affect the precolonial, colonial, and postcolonial world's experience with justice and injustice?

---

The lease on Hong Kong makes no odds to a Syllogism. There is no such thing as postcolonial philosophy. Nor, so far as I am aware, has there ever been a precolonial period in the world's history, or though (different matter) lots of stuff did happen before certain particular west european colonies were founded, often the founding of other colonies by peoples other than west europeans, who came in boats that looked different, or on horses, camels, elephants etc. The happy announcement that we have now reached a post colonial period of the 'world's experience' I greet with raised eyebrow. Try telling this to the tibetans.

David Robjant

back

(83) Jeremy asked:

What is currently considered mainstream philosophy and where is it predicted that philosophy will take us in the coming years?

---

1. depends who you ask

2. either:
(a) the same place we are now, or
(b) somewhere different

You question presupposes that Philosophy is either like Fashion (in which case someone in the know might be able to spot a trend for the next six months), or like Science (in which case progress of some sort is bound to occur so long as we continue to confront reality). Well, I suppose it is a bit like fashion, except that a new style comes out every few hundred years rather than twice a week, which makes spotting trends tricky. And of course philosophy is nothing like science at all, because it is perfectly possible, in philosophy, to avoid any confrontation with reality over many hundreds of years. Hence the answer I was forced to give.

David Robjant

back

(84) Grant asked:

This may be an urban myth I was told that a 2nd year psych paper had the following question, to which only one 'correct' answer was received.

Q.

If this is the question what is the answer?

---

If that ever did appear as a question on a 2nd year psych paper then the lecturer in charge of that course should be sacked for wasting our time with nonsense. The correct answer is of course something like 'b*ll*cks!'.

Shaun Williamson

back

(85) Tom asked:

No theory can prove its own Godel statement but...

Can a theory exist that has only a finite number of axioms, that can prove every mathematical statement OTHER THAN the godel statement for that or any other theory.

---

I don't quite understand your question but I will try to answer it. What Godel's proof proves is that in OUR system of mathematics there is no set of axioms that will allow us to derive all mathematical truths from them.

The Godel statement is just an example of a statement that cannot be derived from our usual axiomatic system. We could extend our system so that the Godel statement is included as an extra axiom but then, as Godel demonstrated, it will be possible to construct another statement that cannot be derived from this new set of axioms.

However we can construct a completely decidable system of mathematics if we introduce certain restrictions. I believe that a system of mathematics that does not include multiplication may allow us to construct an axiomatic system that does not allow Godel's proof.

Another way to do this is to disallow self referential statements. In our system of mathematics the sentence 'This sentence contains five words' is a valid mathematical statement. If we were to disallow such statements then Godel's proof would not be valid.

However in the end we have the system of mathematics that we have and that is not going to change. What Godel discovered is that in our system of mathematics the concept of truth and the concept of derivable from a set of axioms are not the same.

Shaun Williamson

back

(86) Jeremy asked:

What is currently considered mainstream philosophy and where is it predicted that philosophy will take us in the coming years?

---

You must be confusing philosophy with some other subject. It has been said that the most influential philosopher in the western world is W. O. Quine (who is now dead). There is no such thing as mainstream philosophy. Philosophers disagree about everything. There are no agreed philosophical truths. There are no agreed philosophical theories. There are no agreed advances in philosophy and no agreed predictions about where philosophy will take us. So far 2500 years of western philosophy has failed to take us anywhere that we can all agree on.

Shaun Williamson

back

(87) Andrew asked:

Can you point me in the right direction to answer the following. I exist. There is no way that I can logically see that 'I' could not exist. To believe other would be to accept that there are entities that do not exist, which I do not see as possible. My question is then, does this mean that I had to exist? That is not a matter of incredible chance but a certainty?

---

Well of course there are entities that do not exist. Flying pigs do not exist. Given that you exist then of course it is logically certain the you exist. However don't confuse logical certainty with other sorts of certainty. If your parents hadn't met and if your parent hadn't had a sexual relationship then you wouldn't exist and all the logic in the world won't get you around that.

You existence is contingent not necessary. Logical truths are necessary truths, factual truth, including your existence, are contingent.

You are confused about the difference between 'I cannot logically see that I do not exist' and 'I cannot logically see that I could not exist'.

Shaun Williamson

back

(88) Debra asked:

Why do wives go back to abusive husbands?

---

Maybe you should ask a psychologist who has made a special study of the subject to answer that question.

However the simple answer is that these women have low self esteem. They think they don't deserve any better and that they deserve what they get.
Strangely enough the men who abuse them also suffer from low self esteem.

However the more complex answer is that different people often have different reasons for the things that they do. Here we have an evil situation and how conscious the participants are of their own moral responsibility for entering into and staying in this evil situation is not something that can be decided by a simple formula.

The bully and the victim often seek each other out. I wish it wasn't like that but often it is.

Shaun Williamson

back

(89) Donovan asked:

The last few years of my life , i'm now 27, I suffer from indecisiveness. it seems that I find it very hard to make decisions that effect my life (think of career, relation but even just what to make for supper?. on the other hand I have no problem giving people in my surrounding good advice which they , most of the time find very helpful.

how can one decide wether to follow his feelings or his reasons, the hart or the mind.

---

Chronic indecisiveness can be a sign of anxiety. You need to decide how serious your problem is and if you should seek some sort of counselling. Anxiety is generally caused by unresolved problems with regard to work or relationships.

Start by deciding how serious your problem is. If you decide it is serious then get some help.

Shaun Williamson

back

(90) Jim asked:

A child once asked me what 'time' was...and I was stumped...That boy made my life nightmare of reading...and I am still stuck for a good answer

---

You are just confused because although you know what time is, you cannot think of a neat definition to sum up all the diverse aspects of time.

Not everything has a neat definition. Suppose the child had asked you what mathematics was or what a game is, you might find it equally difficult to come up with a neat definition of these things.

Shaun Williamson

back

(91) Louie asked:

Is it possible that there could be a overwhelming stream of information,via the five senses,that bombards us through out our daily routine;this stream cannot all be deciphered/ processed, so eventually the subconscious brain acts as a filter.Filtering out the 'useless' and processing the 'useful'.And that all humans are innately desensitized to there environment

---

There is bound to be some truth in what you say. The brain filters out the things that are unimportant and concentrates of what is new and unknown.

For example the brain is very good at filtering out a constant background noise such as the noise of traffic in a large city. I don't know that desensitized is the best word to use.

However I am not sure that phrases such as 'the subconscious brain' are useful here. After all I am conscious, my brain isn't conscious or unconscious. It is just a brain. I may need a brain in order to be conscious and I have no doubt that my brain filters out irrelevant information but is doesn't do this consciously or unconsciously, it just does it.

Shaun Williamson

back

(92) Louie asked:

We exist; given

We have an unique perspective; given

Everything that has existence, exists. Definition And everything that exists has an unique perspective.(far fetched)

Everything that we perceive exists.

Though it might be impossible for a human to fathom because of the cheer differences in the perspectives,but if you view life as a creation of sorts; then we are no more the perspective of a created object.And if we can create objects in the computer world they too technical should have perspectives.We would never know

---

Louie I think you are speculating too much here. As Wittgenstein said 'Philosophy arises when language goes on holiday'. By this he meant the when we are just thinking about things outside of any guiding context then all sort of things start to seem possible.

Things that we can never know about are the same as things that don't exist e.g. invisible, intangible footballs are non existent footballs and the perspective of objects in computers are no perspective at all. Everything that exists doesn't have a perspective you need to be a conscious being, at the very least, in order to have a perspective. Try substituting the work 'outlook' for the work 'perspective'. It is a word that is less likely to lead you astray.

Shaun Williamson

back

(93) Jim asked:

A child once asked me what 'time' was...and I was stumped...That boy made my life nightmare of reading...and I am still stuck for a good answer.

---

Time is a sequence of relations, specifically the temporal relations that have terms labeled 'before' and 'after'. For example, 'I was born before my sister,' where terms of the 'before' are 'I' and 'my sister.' This is not a great deal of help unless you know more about relations, and relations have given philosophers difficulty for all of recorded history — so much so that many major philosophers have denied their existence altogether.

One of the problems is the question of whether they can be perceived or not. Consider the relation 'on', when your shoe is on your foot: you must be able to perceive this 'on', since otherwise how could you know whether or not the shoe was on your foot? But although you can see, touch, etc. both the shoe and your foot, you cannot see, touch, etc. the relation on: it has no colour, hardness, taste, etc., no perceptible properties at all, in which case how can you perceive it?

Another problem is that relations seem to have an inferior kind of existence to concrete properties: if you take your shoe off your foot the relation on ceases to exist, while the shoe and your foot continue to exist, and this suggests the unreality of relations. A third problem is their extravagant multiplication. Consider the relation 'similar to,' as in 'this is similar to that.' We can say that the 'this' is a term of the relation 'similar to,' and so is the 'that.' So there seem to be two more relations, two 'terms of.' And these have two terms each, and so on ad infinitum.

(If you ever did any set theory in the New Math, you may remember that a relation is defined these days as a subset of a Cartesian product, and the subset is defined by means of a polyadic predicate. Superficially this solves these problems, by making relations be logical constructs, out of their terms. But this is pretty useless. A Cartesian product can only be defined by means of prior relations, such as set membership and subset, and 'polyadic predicated' is only another name for a relation.)

My own view (although many philosophers will disagree with this) is that relations are genuine, real existents but that they are abstract entities — which is why they have no concrete properties such as colour or hardness. And their existence is not inferior because they come into existence — they emerge — just as easily as they go out of it. And, thirdly, some relations have nominal existence only, including those that multiply extravagantly; thus 'term of' is not a real relation, so does not multiply except ion in language. You might like to consider questions such as 'What are space, causation, change, process, identity, similarity and difference?' They are all relational. I am sorry if all of this is somewhat technical, but if you can handle that you may want to look at my book 'Relation Philosophy of Mathematics, Science, and Mind.' You can download it for free from www.sharebooks.ca.

Helier Robinson


back

(94) Alenorme asked:

I was reading Monadology and the idea of a Monad as Leibniz describes it it's very similar to the idea of (super)string as some physicists describes them.

I am not a physicist but I am interested in physics and the concepts it generates. Because I do not have the appropriate knowledge of particle physics and string theory all I can do is speculate. nevertheless this similarity seems to me at the moment very interesting and I want to know

Is it just me or are there some similarities?

---

Yes there are some similarities, such as each being the most basic 'stuff' of theoretical reality, but there are major differences as well. For Leibniz, relations are merely entia rationes, things of the mind. So there is no real space and monads have no size and no location; they are real substances, each with an infinity of attributes. Strings, on the other hand, exist in space (10-dimensional space) and have length. But the basic difference is in what you consider to be the rationality of reality. For Leibniz it was the logic of Aristotle, which is a subject-predicate logic, to which correspond the substances and attributes of his theoretical reality. For modern physicists the rationality of reality is mathematics, to which correspond the relations of reality. Modern science has no use for substances and attributes, except in chemistry, where the concept of substance is not Aristotelian.

Helier Robinson


back

(95) Ellie asked:

what came first egg or the chicken?

---

The egg. Chickens evolved from dinosaurs, which existed long before chickens, and which laid eggs.

Helier Robinson


back

(96) Tom asked:

No theory can prove its own Godel statement but...

Can a theory exist that has only a finite number of axioms, that can prove every mathematical statement OTHER THAN the godel statement for that or any other theory.

---

Godel's two theorems apply to every theory large enough to contain Whitehead and Russell's Principia Mathematica. So the answer to your question is no, provided that Godel is correct. But there are two challenges to Godel. One is that his proof uses self-reference, which is a notorious source of paradox. The other is that it presupposes some very large numbers, and this presupposes no limit to the size of numbers — i.e. an infinity of numbers. It may just be that there is no such thing as infinity, in which case Godel's theorems may be invalid. (If there is no infinity then there must be a largest finite number. Any 'larger' numbers cannot exist except in language, and some Godel numbers may be such.)

Helier Robinson


back

(97) David asked:

I have two questions regarding the cosmological argument for the existence of God. The argument has been presented to me as follows:

1. Everything that begins to exist has a cause

2. The universe began to exist

Therefore the universe has a cause.

And, of course, that cause is God. First, why does that 'cause' automatically have to be a 'God' in the sense that we usually think of God (i.e., having a personality, all-powerful, etc.)
Secondly, premise 1 seems to be true because of our observations of the universe. When we look around, we notice that everything in the universe that begins to exist does indeed (seem to) have a cause. But every effect in the universe seems to have a physical cause. In other words, based on our experience we should word reword premise 1 as 'Everything in the universe that begins to exist (i.e. something tied to the physical universe) has a physical cause.' In which case the conclusion wouldn't make sense unless the cause itself is also physical.

By physical, I don't mean something that can necessarily be touched or something. For example, if love between two people begins to exist, that love isn't 'physical,' but it is certainly preceded by physical causes, such as brain chemistry.

Anyway, is this a valid critique of the argument?

---

First of all, the argument depends on what definition of God you use. Various definitions are: all powerful, all loving, all knowing, prime mover, first cause, or some or all of these. Most theologians include all of these, in which case the first cause is God. Secondly, there is a fallacy in your presentation: the fallacy of composition, which is the fallacy of supposing that what is true of the parts is true of the whole; for example, an army is composed of men therefore the army is a man. You say that everything in the Universe that begins to exist has a physical cause; but it does not follow that the universe as a whole, which began to exist, has a physical cause: it might or it might not. One argument against the first cause argument is that if the Universe had to have a cause (a Creator) then what caused the Creator? If the reply is that the Creator was causa sui (self-caused) or the Creator existed for ever (eternity) then you can say either of these of the Universe as a whole. Another argument is that the principle that everything has a cause is as much in need of proof as is the existence of God.

Helier Robinson


back

(98) Andrew asked:

Can you point me in the right direction to answer the following. I exist. There is no way that I can logically see that 'I' could not exist. To believe other would be to accept that there are entities that do not exist, which I do not see as possible. My question is then, does this mean that I had to exist? That is not a matter of incredible chance but a certainty?

---

It is indubitable, for you, that you exist because you have to exist in order question you existence. But because your is certain it does not follow that the cause of your existence is certain — that is, certain that you has to exist. You do not have certain knowledge about the cause of your existence, as is shown by the possibility that you were created five minutes ago, complete with all you memories, as well as the possibility that your existence was uncaused.

Helier Robinson


back

(99) Lauren asked:

If you force sex upon a prostitute, is it rape... or stealing?

---

That would be rape, Lauren: having sex with someone without their consent is rape, whether or not they consent to sex on other occasions in exchange for cash. (I doubt forcing someone to do something could count as 'stealing' in any case, because there is no item of personal property that is being stolen. But that's a legal issue.)

Your question raises the issue of whether raping a prostitute is somehow less bad than raping a non-prostitute — to which the answer is definitely 'no'! If someone jumps in your car and makes you drive them somewhere with a knife to your throat, it makes no moral difference whether or not you happen to be a taxi driver!

Helen Beebee

The British Philosophical Association
http://www.bpa.ac.uk

back

(100) Scott asked:

Who is more intelligent?

A. A person who knows everything. is A (Impossible?) Doesn't it depend on your environment? Student asked this question, I don't have a response for this HELP!! right and wrong?

B. A person who know he doesn't know everything.

I am teaching world history and my students attained this question from a Socrates quote: 'I am the wisest man in the world, for I know one thing, and that is I know Nothing.' What is your opinion? and Do you mind if I share it with my students?

---

Well, (B) sets the bar pretty low! Nobody knows everything, and surely pretty much everybody knows this about themselves. (Nobody apart from me knows how many pens there are on my desk right now, for example. Actually my desk is quite messy so not even I know that.) You'd really have to be quite dense, or perhaps deluded, to think you know everything! I suppose someone who just hadn't even considered whether or not they know everything might count as someone who fails to satisfy (B). But I'd say that they're just lacking in imagination rather than lacking in intelligence.

I'm not sure whether (A) is strictly possible. This is a big issue in the philosophy of religion — is omniscience really possible? (Can God know what it's like to stub your toe or have a hangover?) But it's certainly impossible for mere human beings! Nobody can know the full decimal expansion of pi, because it's infinite. And in practice, there are lots of things that nobody will ever know. (How many water molecules are there in the glass on my desk? Exactly how many hairs were there on the head of the person who first uttered the word 'cat'?)

So your question, as stated, seems a bit redundant, because human being in fact does or ever will, or indeed could, know everything. Maybe some hypothetical infinite super-being could know everything. I guess such a being would be more intelligent than someone who knows they don't know everything, just because, as I said, that sets the bar so low.

A more down-to-earth question in the same ballpark, though, would be whether knowing lots of stuff (not everything!) makes you more intelligent than appreciating that your knowledge of the world is in fact extremely limited (which, for all of us, it is). I'd say not. Being able to learn and recall lots of facts isn't the same as being intelligent. (Not that I am going to try and give you a definition of intelligence!)

Go ahead and share with your students if you think it'll help!

Helen Beebee

The British Philosophical Association
http://www.bpa.ac.uk

back

(101) Jim asked:

what is the philosophical definition of 'time'? It may seem simple but try to give an answer to a bright 10 year old boy..

He can read a clock...He wants a simple explanation of 'time' if there is one...I have come to the belief there isn't

Geographically, us rounding the sun, is only a definition of the measure of time...

---

You are right, Jim, to distinguish between a measure of time and time itself. And you will have noticed that a creative refusal to make exactly this distinction is one element in GK's delightful answer.

His thought that clocks are to time as a ruler is to length helps is to an image, for which we are grateful, and with that the image strikes one as somehow explanatory — but how explanatory is it? If time were indeed just whatever it is that a clock measures, there would not be nothing to time but hours, minutes and seconds. Yet there is some 'before' the second hand moves, and some 'after', just as there are innumerable lengths fractionally larger than 2 inches, or any other measured length. If this is so for any measurement of anything at all, why would one confuse the measure with the measured?

Because knowing the time or place accurately sometimes matters we find ourselves forced to refine and improve our measuring devices (our digital stop watches, vernier scales and micrometers) in pursuit of a reality that lies beyond the names we have for the days of the week, or even beyond the hours of the day. Week next friday will not do for some purposes, and sometime after ten won't do for others. That such refinement of our terminology may be necessary, or even possible, shows the existence of some reality outside our current apparatus — or we would still be stuck with the sand clock.

But if this refinement is understood as a pursuit of perfect accuracy, it must go with the reminder that a final arrival is something utterly inconceivable. We invented the hour, then the minute, and the second, and now nano-seconds. When the hour was the latest invention, no-one was tempted to take the measure for the real thing, and to imagine and understand time we flew to other images, of flowing rivers, water under the bridge, birds in flight, grim reapers and so on. Now the better measuring devices can tell one fraction of a second from the next, and we are tempted to think that we've finally caught the thing itself in our clockish net — for surely there can be no further fractions? But naturally there can. And as for catching time itself in our measuring, well, on that point we have, for all our nano-seconds, come not one jot onwards from the sand-clock. An infinity of future refinement lies before us now, just as surely as it did then.

Another point obscured by the thought that time is to a clock as length is to a ruler, is that time has a direction, and cannot be traversed or completed in any number of directions, as with a line on a piece of paper. Oh yes, I know that time has been called the fourth dimension, and that understanding it so makes some equations in physics come out nicely. How pretty for the physicists. But in so 'refining' the concept of time physicists have in fact exchanged that concept for something else entirely. What we mean by 'time' is an irrevocable, continuous and involuntary going forward, something essential to the character of all human experience whether spatial or not. 'Months went by' as Seagoon used to say, 'I couldn't stop them.' Here we are back with images. The arrow in flight. The river flowing irresistibly under the bridge. These metaphors are truer to time than any clock or algebra.

David Robjant

back

(102) Lauren asked:

If you force sex upon a prostitute, is it rape... or stealing?

---

There is nothing in morals or in the criminal code to say that if you commit one sin or offence, then by that fact you cannot also be committing another. Possibly both rape and stealing, then, on that account.

However, and this is a legalistic rather than a philosophical point, I am under the impression that in english law at least one cannot be said to own one's own body, so to speak. I believe that there was originally a basis or support for this principle in christian theology: your body belongs to no one but God — this featured in the 19th century agitation against slavery. The many varieties and degrees of assault and injury can, therefore, none of them be described in law as cases of theft even where body parts or life itself have been taken. Thus in the case of 'stealing' a kidney the law would speak not of theft but rather of grievous bodily harm, or some such. In general, and with renewed cogency after the abolition of slave owning as an offence against God, where persons are involved the law speaks not of various kinds of theft but rather of kidnapping, rape, GBH, Murder etc.

And quite right too.

What confuses one is the thought that the prostitute is, as the phrase has it, 'selling her body', which implies, in violation of christian theology and english law, that she owns her own body. It implies other things, too, in violation of common sense — see 'you sell it, and you still got it'. There are other ways of describing the situation, I take it. I suspect that, where it considers the matter, the Law rather enjoys talk of selling 'services'. Has the rapist obtained 'services' by force, and in that way committed a variety of theft? Hm. There would be two ways to go here, and my sketchy understanding of the law gives out completely here. If compelled to join a 'moot' my line would be that while paying for such services with monopoly money would be a kind of theft (i.e. fraud), in the difference case of violence or threat of it leading to sex that compulsion would make it impossible for there to be any commercial exchange here and, by that fact, impossible for there to be any 'services'. That is, there would just be rape, and no theft.

David Robjant

back

(103) Andrew asked:

Do physicists believe in something that has no extension in space and cannot be detected.

I don't know I am not a physicist but here's a thought experiment.

Physics makes quantum leaps in its knowledge base and predictive power. More illogical facts are held to be true. As the single particle of light that goes through two different slits at the same time is held to be true now.

The explanatory power of physics become exponentially greater than it is now. The only problem is that it does not add up; there is a gap in the maths. They are forced to call this gap in the maths "dark matter". In order to carry on their work they have to factor "dark matter" into their equations. Nothing works without it. But they do not know where dark matter is. It cannot be detected. Why should they say it has extension.

Does it exist? If so what is it? is it a nonphysical thing. It certainly affects physical things?

---

You need to think carefully about modern physics and keep in mind that physicists are scientists so they do not ask us to believe in illogical facts nor do they believe in matter that has no extension in space.

You have to remember that sub-atomic particles are not things in the ordinary sense and do not have to behave in the way that material things do. This may make the sub atomic world impossible to picture but it does not make it illogical.

If a single photon can pass through two slits at the same time then this simply shows us that a photon is not a small billiard ball and it is nothing like a billiard ball or any other thing that we are familiar with.

Astrophysicists introduced the concept of dark matter because the best estimates of the amount of detectable matter in the universe do not explain the speed of expansion of the universe. The idea of dark matter is that it is matter that does not emit any detectable radiation e.g light, heat, x-rays or neutrinos etc. However its presence is detectable because like all matter it has mass and therefore exerts a gravitational force. Astro- physicists do believe that they can detect concentrations of dark matter in certain regions of space, so it is not true to say that they do not know where it is.

However Astrophysics is still in its infancy and there are many things we do not yet understand. We still have not managed to integrate Einstein's theory of relativity with quantum mechanics. Theories about dark matter are provisional are likely to change in the future.

Shaun Williamson

back

(104) Scott asked:

Who is more intelligent?

A. A person who knows everything. is A (Impossible?) Doesn't it depend on your environment? Student asked this question, I don't have a response for this HELP!! right and wrong?

B. A person who know he doesn't know everything.

I am teaching world history and my students attained this question from a Socrates quote: "I am the wisest man in the world, for I know one thing, and that is I know Nothing."

What is your opinion? and Do you mind if I share it with my students?

---

As a simple matter of logic a man who knows everything knows more that a man who knows that he does not know everything.

Intelligence is related to problem solving ability and cannot be directly related to how much you know.

It is extremely unlikely (in fact I would say impossible) that any one individual could know everything.

Socrates idea that he knew nothing is of course nonsense because he knew his own name and where he lived. Like many philosophers he was inclined to overlook the importance of our knowledge of such simple facts.

Shaun Williamson

back

(105) Sara asked:

Is it possible for nothing or nothingness to exist?

---

As Wittgenstein pointed out it is a primitive superstition about language that leads us to think that all words are somehow the names of things. So if a word like 'nothing' does not name a physical thing it must name some ghostly thing such as 'nothingness'.

'Nothing' is a word in our language and it is a very useful word. You can ask me 'How much money do you owe me?', and I can answer 'Nothing'. Surely it is clear here how the word functions and how useful it is. There is no reason to suppose that I am claiming that you owe me some ghostly amount of money. I could equally well have answered, 'You don't owe me any money'.

Shaun Williamson